geometría.pdf

March 27, 2019 | Author: Jhonatan Sorto | Category: Triangle, Circle, Rectangle, Tangent, Classical Geometry
Share Embed Donate


Short Description

Download geometría.pdf...

Description

curso curso de post-g post-grad rado o para profe profesor sores es ´ tica especialid especialidad ad en matem matematica a

GEOMETR´IA EUCLIDEANA Equipo de Dise˜ no: no: Nahomy Jhopselyn Hern´  andez Cruz  Gabriel Alexander Chicas Reyes  Eduardo Arnoldo Aguilar Ca˜  nas  H´ ector ecto r Enmanuel Enma nuel Alberti Arroyo Ernesto Am´ erico erico Hidalgo Castellanos  Castel lanos  Juan Jua n Agust´ın ın Cuadra  Cua dra  Claudia Patricia Corcio L´  opez de Beltr´  an  Carlos Mauricio Canjura Linares  Oscar Armando Hern´  andez Morales  Aar´  on Ernesto Erne sto Ram´ Ram´ırez Flores 

5 de abril de 2010

´Indice ´ ngulos entre paralelas. 1. A

2

2. Tri´ angulos: Teoremas Fundamentales.

7

3. Congruenci Congruencia a de Tri´ angulos.

16

4. Cuadril´ Cuadril´ ateros: ateros: Clasificaci´ on y Propiedades.

24

´ ngulos en la Circunferencia. 5. A

30

6. Teorema rema de Thal Thale es y su rec´ ec´ıpr proc oco. o. Sem Semejanz janza a de Tri´ ri´ angul ngulo os.

46

7. Puntos y Rectas Notables del Tri´ angulo.

58

8. Soluci Soluci´ o ´n a Problemas Selectos.

72

1

1.

´ Angulos entre paralelas.

´ ANGULOS Definimos como angulo a´ngulo a la figura geom´etrica etrica formada por dos rayos (o semirrectas) distintas que tienen el mismo origen. Ese origen se llama   v´erti . Al ´angulo ang ulo de v´ertice erti ce O y er tice ce del de l angulo ´  rayos OA y OB  se le denota ∠AOB . BO C  son  adyacentes  si y s´olo  O B  y los lados Dos angulos a´ngulos ∠AOB y ∠BOC  olo si tienen un lado com´ un un  OB no comunes  OA  O A  y  OC   O C   est´ an en semiplanos distintos, determinados por el lado com´ an un. un.

angulo es la semirrecta que lo “divide” en dos ´angulos angulos adyacentes iguales. Bisectriz  de un ´angulo Dos angulos a´ngulos son: Congruentes o Iguales : si tienen igual medida. Suplementarios : si su suma es 180 . Complementarios : si su suma es 90  .

Por otra parte, dos rectas en el plano pueden ser  secantes  o  paralelas ,1 dependiendo si se cortan o no; adem´as, as, si las rectas son secantes, el punto de corte es ´unico, unico, y definen cuatro ´angulos, angulos, que se agrupan por parejas en ´angulos angulos  opuesto   (las parejas de ´angulos angulos tales que  opue stoss por el v´ ertice  ert ice  (las uno est´a formado por la prolongaci´ on on de los lados del otro). Los angulos a´ngulos opuestos por el v´ertice ertice son iguales (Justifique), p or lo que dos rectas secantes forman cuatro ´angulos angulos que definen dos parejas de ´angulos angulos iguales, y si tomamos un miembro de cada pareja, se tienen dos ´angulos angulos suplementarios. En particular, si las rectas son secantes y forman cuatro ´angulos angulos iguales, ser´an an llamadas   rectas perpendiculares ,2 y los ´angulo ang uloss as´ as´ı genegen erados son llamados angulos ´  rectos . Y como es muy conocido, un angulo ´  agudo   es aquel cuya ´  obtuso  es aquel cuya medida es  mayor medida es  menor  a la de un ´angulo angulo recto, y un angulo que un ´angulo angulo recto; en particular, un angulo ´angulo obtuso ser´a llamado angulo ´  llano  si su medida es el  doble  que la de un ´angulo angulo recto.

´ ANGULOS ENTRE PARALELAS Al intersecar un par de rectas paralelas por una recta llamada  transversal  o  secante , se forman los siguientes tipos de ´angulo: angulo: ´  a´ngulos no adyacentes situados en el mismo lado de Angulos Correspondientes : Son dos angulos

la secante, uno en el interior y otro en el exterior de las paralelas. ´  Angulos Alternos Internos : Son dos ´ angulos no adyacentes situados en el interior de las angulos

paralelas, y en distintos lado de la secante. 1 2

Si la recta  AB  es paralela a la recta  C D, se denota  AB CD . Si la recta  AB  es perpendicular a la recta  C D, se denota  AB CD .

 

2

 ⊥

´  a´ngulos no adyacentes situados en el exterior de las Angulos Alternos Externos : Son dos angulos

paralelas, y en distintos lado de la secante. ´  angulos no adyacentes situados uno en el interior y el otro angulos Angulos Conjugados : Son los ´

en el exterior de las rectas paralelas y del mismo lado de la secante. Las propiedades fundamentales de los ´angulos angulos entre entre paralelas paralelas son: 1. Los angulos a´ngulo s correspo co rrespondientes ndientes son iguales entre s´ s´ı. 2. Los angulos a´ngulos alternos internos son iguales entre s´ı. ı. 3. Los angulos a´ngulos alternos externos son iguales entre s´ s´ı. 4. Los angulos a´ngulos conjugados son suplementarios.

´ Figura 1: Angulos Angulo s entre entre las rectas rectas paralelas paralelas L1 y L2 .

Ejercicios 1. Tres ´angulos angulos adyacentes forman un semiplano y tienen sus medidas proporcionales a los n´umeros umeros 5, 7 y 8. Hallar la medida del menor ´angulo. 2. Demostrar que las bisectrices de dos ´angulos angulos suplementarios son perpendiculares.

3. En la figura adjunta, L1 L3 L4. Calcular  x .

 

L

2

y

3

4. Con ayuda de la figura 2, demuestre que: Si L1

  L   entonces γ  = α + β . 2

Figura 2 5. En la figura 3, AB

  F G. Hallar el ´angulo  x  si el ∠AM F  = 90

y el

∠M AB  =

110 .

Figura 3 6. Calcular el ∠OP Q, si OP  es bisectriz del ´angulo  O ,  L 1

  L

Figura 4

4

2

y  P Q

⊥ L . Ver figura 4. 1

7. En la figura 5, L1

  L

2

y L3

  L , calcular α. 4

Figura 5 8. En la figura 6, calcular x, si  L 1

  L . 2

Figura 6

9. Calcular la medida θ  del gr´afico anexo, si las rectas L1 y L2  son paralelas.

5

10. En la figura 7, L1

  L

2

y L3

  L . Hallar el valor del ´angulo  θ . 4

Figura 7 11. Sea ∠AOB = 24  , en la regi´on exterior a dicho ´angulo se traza el rayo OC. Hallar la medida del ´angulo formado por las bisectrices de los ´angulos AOC  y BOC . 12. Del gr´afico 8, calcular  y , cuando  x  tome su m´aximo valor entero.

Figura 8

6

2.

Tri´ angulos: Teoremas Fundamentales.

´ TEOREMAS FUNDAMENTALES EN TODO TRI ANGULO. Diremos que tres puntos que pertenecen a una misma recta son  puntos colineales ; de manera an´aloga, si tres rectas pasan por un mismo punto, ser´an llamadas rectas concurrentes . Si tomamos “al azar” tres puntos en el plano, en muy raras ocasiones estos puntos estar´an alineados,3 y diremos entonces que son los v´ertices de un tri´angulo; an´ alogamente sucede con las rectas, tres rectas por lo general no concurren, y la figura geom´etrica que ´estas definen es tambi´en un tri´angulo.4 Una definici´on completa para nuestros intereses es la siguiente:

Definici´ on de Tri´ angulo.  Si  A ,  B y C  son tres puntos cualesquiera no colineales (Ver figura 9), entonces la reuni´on se los segmentos AB , BC  y AC   se llama tri´angulo ABC  y se denota por ABC . Los puntos  A ,  B y  C  se llaman  v´ertices  y los segmentos  AB ,  B C  y  AC  se llaman lados . Simb´ angulos  olicamente: ABC  =  AB BC  AC . Todo tri´angulo  ABC   determina tres  ´  a angulo internos o interiores : ∠ABC , ∠ACB y ∠BAC , y se llamar´ ´  externo o   exterior , al ´angulo determinado por un lado y la prolongaci´ on del lado adyacente, en la figura 9, α, β  y θ son ´angulos exteriores.









Figura 9: Elementos del Tri´angulo Dado el ABC , se tiene que AB  +  BC  +  CA  =  p  = 2s, donde p  es llamado el per´ımetro y s el semiper´ımetro del tri´angulo. Para abreviar, suele asociarse a cada v´ertice un lado opuesto, y viceversa, por ejemplo, el lado opuesto de A es BC , y es frecuente que se denote por a; an´alogamente b  =  C A, c  =  AB .

 

Teorema 1:   En todo tri´a ngulo, la medida de un a´ngulo exterior es igual a la suma de las medidas de dos a´ngulos interiores del tri´angulo no adyacentes a ´el. La demostraci´on de este teorema se basa en las relaciones de ´angulos entre paralelas; se deja al lector que haga la demostraci´on (Sugerencia: por un v´ertice, trace una recta paralela al lado 3

En teor´ıa de probabilidades, ¡la probabilidad que esto ocurra es cero! El t´ermino m´as riguroso para esta figura es  tril´  on: si atero. En este caso, habr´ıa que hacer una consideraci´ hay un par de rectas paralelas, el tril´atero definido ya no es “normal” seg´un nuestro sentido com´un, sin embargo, ¡sigue siendo un tril´atero! 4

7

opuesto)

Corolario:  En todo tri´angulo, la suma de las medidas de sus tres ´angulos internos es igual a 180 . Teorema 2: Desigualdad Triangular.  En todo tri´angulo, la longitud de uno de sus lados est´a comprendido entre la suma y la diferencia de los otros dos. Sin ser muy rigurosos, suponga que dado el segmento AB  se traza con centro en A   una circunferencia de radio r1 , y con centro en B  una circunferencia de radio r2 ; si   AB < r1  +  r 2 , las circunferencias se cortar´an en dos puntos, y cualquiera de ellos puede ser el v´ ertice C , as´ı AB < BC  + CA; en cambio, si AB =  r 1  + r2  o peor a´un, si  AB > r1  + r2 , la construcci´on del ABC  no es posible.

 

La Desigualdad Triangular es un resultado fundamental, a partir de ´esta y de su modelo de demostraci´ on se generan los   Criterios de Congruencia de Tri´  angulos ; a groso modo, si dadas ciertas condiciones, la construcci´on de una figura geom´etrica (un tri´angulo en particular) queda determinada de manera u ´ nica, entonces dos figuras que reunen las mismas condiciones ser´an llamadas  figuras congruentes . As´ı, si se tienen tres segmentos (cuyas longitudes cumplen la desigualdad triangular), dejando uno fijo y construyendo las circunferencias con centros en los extremos de este segmento y radios las longitudes de los otros segmentos, por construcci´on, s´olo ser´a posible obtener dos tri´angulos (uno con cada punto de intersecci´on de las circunferencias), que son b´asicamente el mismo pero la orientaci´on de los ´angulos es contraria; as´ı, si se sabe que dos tri´angulos cumplen tener lados respectivamente iguales, por construcci´on, deben de ser iguales. Este es el conocido criterio LLL de congruencia de tri´angulos; m´ as adelante se detallar´an el resto de criterios, pero a partir de este probaremos el siguiente resultado:

Teorema 3:   En todo tri´angulo, se cumple que a lados iguales se oponen ´angulos iguales, y viceversa. Suponga que ABC  es tal que AB = AC , entonces, por criterio LLL, ABC   es congruente al ACB   (en ese orden, porque AB = AC , BC  = CB y CA = BA ), entonces, los ´angulos que se oponen a los ´angulos iguales son iguales. Para el rec´ıproco necesitamos otro criterio de congruencia, por lo que la demostraci´on se dejar´a incompleta; retome esto en la secci´o n de congruencia de tri´angulos.

 

 

 

Teorema 4:  En todo tri´angulo se cumple que a mayor lado se opone mayor ´angulo y viceversa. Este teorema se deja como ejercicio para el lector (Sugerencia: utilice el teorema anterior, tome el lado mayor y defina un punto adecuado que genere un tri´angulo con dos lados iguales.)

´ DE TRIANGULOS. ´ CLASIFICACION 1. Con relaci´ on a sus lados: 8

a )   Escaleno : si sus tres lados no son congruentes. b ) Is´  osceles : si por lo menos dos de sus lados son congruentes. c )   Equil´  atero : si sus tres lados son congruentes (note un tri´angulo equil´ atero es tambi´en

is´osceles, y que los tres ´angulos internos son iguales entre s´ı e iguales a 60) 2. Con relaci´ o n a sus ´angulos internos: angulo mayor es agudo (note que entonces los tres ´angulos son a )   Acut´  angulo : si su ´ agudos) angulo mayor es ´angulo recto (note que el ´angulo en cuesti´o n es b )   Rect´  angulo : si su ´ u ´ nico y que los otros dos ´angulos son agudos; as´ı, en un tri´ angulo rect´angulo, la hipotenusa es mayor a los catetos) c ) Obtus´ angulo, si el a´ngulo mayor es a´ngulo obtuso (note que el ´angulo en cuesti´on

es u ´nico y que los otros son agudos; as´ı, en un tri´angulo obtus´angulo, el lado que se opone al a´ngulo obtuso es el lado mayor)

´ L´INEAS NOTABLES EN UN TRI ANGULO. 1.   Altura:  Se llama altura de un tri´angulo al segmento que parte de uno de sus v´ ertices y llega en forma perpendicular al lado opuesto o a su prolongaci´on. 2.   Mediana:  Se llama Mediana al segmento que une un v´ertice con el punto medio del lado opuesto. 3.   Mediatriz:  Se denomina mediatriz de un lado de un tri´angulo es la recta perpendicular a dicho lado en su punto medio. 4. Una  Bisectriz:  La bisectriz es la recta que “divide” en dos ´angulos iguales a un a´ngulo dado; en particular, es   bisectriz interna   si es la bisectriz de un ´angulo interno de un tri´angulo, y  bisectriz externa  si es la bisectriz de un ´angulo externo de un tri´angulo.

DISTANCIA DE UN PUNTO A UNA RECTA. En la figura 10, sea P  un punto exterior a una recta L, la longitud de la perpendicular P M  a la recta L  es la  distancia del punto P   a dicha recta . Esta perpendicular tiene la propiedad de ser u ´nica y su longitud es la distancia m´ınima del punto a la recta (Pru´ebelo utilizano el hecho que la hipotenusa es mayor que los catetos). Los segmentos  P A  y  P B  no son perpendiculares a L  y se llaman oblicuas.

´ TEOREMA DE PIT AGORAS. Abordamos el estudio de las   Relaciones M´etricas , del cual solo realizaremos el an´alisis del famoso  Teorema de Pit´  agoras , cuyo enunciado es el siguiente:

Teorema: Pit´ agoras.  En un tri´angulo rect´angulo, el cuadrado de la hipotenusa es igual a la suma de los cuadrados de los catetos.

9

Figura 10 Una demostraci´on de este teorema es debida a  Thabit ibn Qurra   (836-901), la cual consiste en diseccionar la figura que se forma al construir dos cuadrados de lados respectivamente iguales a los catetos de un tri´angulo rect´angulo, como se muestra en el gr´afico 11.

Figura 11

Rec´ıproco del teorema de Pit´ agoras:  Si en un tri´angulo el cuadrado de un lado es igual a la suma de los cuadrados de los otros dos lados, el tri´angulo es rect´angulo.5

Ejercicios.

1. En la figura adjunta ambos tri´angulos son equil´ateros. Encuentre el valor de ϕ.

5

Ver demostraci´ on en la secci´on de congruencia de tri´angulos.

10

2. En la figura 12, calcular el ∠x  si el

∠AOB  =

100 y L1

  L . 2

Figura 12 3. (*) En la figura 13,  ABDE   es un cuadrado y BC D  es un tri´angulo is´ osceles con BD = DC . Si ∠ABC  = 160  , determinar la medida de ∠AEC .

Figura 13

4. (*) (XV Competencia de Clubes Cabri Primera Ronda) En la figura adjunta, ABCD  es un rect´angulo tal que AB = 2 BC . M  es el punto medio de AB  y los tri´angulos AME  y  M BF  son equil´ateros. Si  P  es la intersecci´on de las rectas  DE  y  C F , encuentre los ´angulos del CDP .



5. Si  AB y F G  son rectas paralelas, el 150 . Calcule θ. Figura 14

∠ABC  = ∠CDE  =  θ ,

el

∠DEF 

=

θ 2

y el

∠GF H  =

6. Probar que una bisectriz exterior de un tri´angulo es paralela al lado opuesto si y s´olo si el tri´angulo es is´osceles. 11

Figura 14 7. (*) Hallar la suma de los ´angulos α +  + θ + φ  en la figura 15.

Figura 15 8. Determine el valor de la suma

∠A

+ ∠B  + ∠I  + ∠H  + ∠F  + ∠G. Figura 16.

Figura 16

12

9. En el ABC  el ∠BAC  = 36 y AC  = AB . Probar que la bisectriz interior BD , D en AC , es congruente con el lado  B C .

 

10. Sea ABC   un tri´angulo rect´angulo en B con AB = BC , se construye exteriormente el tri´angulo equil´atero  B CD . Encuentre el ´angulo ∠DAB . 11. En el ABC , AB = AC  y D   un punto sobre la recta AC , tal que BC  = BD = DA . Determine la medida del ´angulo ∠ABD , si:

 

a entre A  y  C . a ) D  est´ a entre  D y  C . b ) A  est´ 12. En un ABC ,  D  es un punto sobre el lado  AC  tal que  AB  =  AD . Si ∠ABC  90  , hallar el ∠CBD .



13. En la figura 17, el

∠ABC  = ∠ACE ,  DC  =  EC ,

− ∠ACB =

¿Qu´e l´ınea notable es AD del

 BC A?

Figura 17 14. Se tiene un tri´angulo is´ osceles ABC , AB =  BC  en el cual se traza al altura AF  tal que BF  = 6 y  F C   = 2. Hallar  AC . 15. ¿Cu´al es el valor de b

− a  en la figura 18?

Figura 18

13

16. La hipotenusa BC   de un tri´angulo rect´angulo ABC   se divide en 4 segmentos congruentes por los puntos G, E  y H . Si BC  = 20, encuentra la suma de los cuadrados de las longitudes de los segmentos AG,  AE  y AH . Figura 19.

Figura 19 17. (*) Dado un cuadrado   ABCD , se construyen los tri´angulos equil´ ateros ABP   (exteriormente) y ADQ  (interiormente). Probar que C ,  P  y Q  est´an alineados. 18. (*) Sea ABC  un tri´angulo rect´angulo con ∠CAB = 90 . D  es un punto sobre la prolongaci´on de BC  tal que BD =  B A. E  es un punto en el mismo semiplano que A  respecto de  B C , tal que CE  BC  y adem´as CE  =  C A. Mostrar que A, D y E  est´an alineados.

 ⊥

19. El cuadril´ atero   ABCD  mostrado en la figura 20 cumple que AB CD y BC  DA .6 Sobre las prolongaciones de AB y AD  se construyen puntos E  y F  tales que BC  =  B E  y DC  =  DF . Demuestre que C , E  y F   est´an alinedos.

 

Figura 20

20. (*) En la figura adjunta, AB = BC  = CD = DE  =  E F  =  F G  =  GA . Calcule la medida del ∠DAE .

6

El cuadril´atero  ABCD  es un paralelogramo.

14



21. (*) (XXVIII Olimpiada Brasile˜ na de Matem´atica) En la figura 21,  AB  =  AC ,  AM  =  AN  y ∠CAM  = 30  , encuentre el valor del ∠BM N .

Figura 21 22. Los lados de un tri´angulo is´ osceles son 12 y 5 metros, ¿cu´al es su per´ımetro? 23. Muestre que los lados de un tri´angulo cumplen que a

 | − b| < c  y que c <

a+b+c 2

.

24. Muestre que es posible construir un tri´angulo con segmentos de longitudes  a ,  b ,  c  si y s´olo existen n´umeros positivos  x , y , z  tales que: a  =  x + y , b  =  y  + z ,  c  =  z  + x. Problemas de Refuerzo.

25. (*) (Etapa semifinal Estatal de XXII Olimpiada Mexicana de Matem´ aticas) En la figura 22 se muestra un hex´agono regular  ABCDEF  de lado 1. Los arcos del c´ırculo que est´an dibujados tienen centro en cada v´ertice del hex´agono y radio igual a la distancia al v´ertice opuesto. P , Q, R, S , T  y U   son los puntos de corte de estos arcos. ¿Cu´anto mide cada lado del hex´agono  P QRSTU ?

Figura 22

15

3.

Congruencia de Tri´ angulos.

CRITERIOS DE CONGRUENCIA. Definici´ on de Congruencia de tri´ angulos.  El ABC  es congruente al A B  C  si: AB = A B  , AC  = A C  , BC  = B  C  , ∠ABC  = ∠A B  C  , ∠ACB = ∠A C  B  y ∠BAC  = ∠B  A C  . Simb´olicamente: ABC  = A B  C  . V´ease figura 23.

 

 

 



Figura 23: Definici´on de Igualdad de Tri´angulos. La definici´on anterior establece que dos tri´angulos son congruentes si tanto los lados como los a´ngulos se presentan en pares respectivos congruentes. Esto, seg´un la visi´on de Euclides, significa que un tri´angulo es posible superponerlo sobre el otro (se puede desplazar, girar o reflejar) y coincidir´a de manera perfecta. Sin embargo, es importante mencionar que en muy raras ocasiones se tendr´ a a disposici´on tanta informaci´on, de all´ı la importancia de los criterios de congruencia, que establecen los requisitos m´ınimos para garantizar que dos tri´ angulos son congruentes. El siguiente es el primero de los tres criterios de congruencia de tri´angulos, y se denomina ´ criterio de LADO-ANGULO-LADO, en s´ımbolos: L-A-L.

Criterio L-A-L.   Si los tri´angulos ABC  y A B  C   presentan las congruencias: AB = A B  , AC  =  A  C  y ∠BAC  = ∠B  A C  , entonces ABC  = A B  C  .

 



Figura 24: Criterio LAL Seg´ un el criterio L-A-L, dos tri´angulos son congruentes si en uno de ellos existen dos lados y el ´angulo (comprendido entre dichos lados), respectivamente congruentes a dos lados y el ´angulo (comprendido entre dichos lados), en el otro tri´angulo.

Criterio A-L-A. Sean ABC  y  A B  C   dos tri´angulos tales que: AC  =  A  C  , ∠BC A  = y ∠BAC  = ∠B  A C  , entonces ABC  = A B  C  .

 



16

∠B

 C  A

Figura 25: Criterio ALA.

Criterio L-L-L.   Si un tri´angulo tiene sus tres lados respectivamente congruentes a los tres lados de otro tri´angulo, entonces estos dos tri´angulos son congruentes.

Figura 26: Criterio LLL. agoras . Ahora demostraremos el  Rec´ıproco del Teorema de Pit´  2 Demostraci´ on: Sea ABC   un tri´angulo talque BC  = AB 2 +  AC 2 , por construcci´o n sea el A B  C    rect´ angulo en A   tal que A B  = AB y A C  = AC , entonces por el teorema de Pit´agoras B  C 2 =  A  B 2 + A C 2 , as´ı que B  C 2 =  BC 2 , de donde B  C  =  BC  y por el criterio LLL, se deduce que el A B  C   = ABC , por lo tanto el ∠BAC  = ∠B  A C   = 90 .



 



TEOREMA DE LA BASE MEDIA En todo tri´angulo, el segmento que une los puntos medios de dos lados es paralelo al tercer lado e igual a su mitad. En la figura 27, M N  es el segmento que une los puntos medios de los lados  AB  y  BC  del

ABC ,

´  a este segmento se le llama  BASE MEDIA DEL TRI ANGULO  . Se verifica que MN  =

que  M N  AC . Demostraci´ on:

 

AC 

2

y

1. Prolongar el segmento M N  hasta el punto P  tal que  M N  =  N P . 2. Los tri´angulos M N B y P N C   son congruentes, ya que BN  = N C , MN  = N P  y el ∠P N C  = ∠BN M , por consiguiente, el ∠N CP  = ∠M BN , por lo tanto,  C P  M B  (Por ´angulos alternos internos iguales). Adem´ as, P C  =  M B =  M A; con lo cual se tiene que: M A  =  P C .

 

17

Figura 27: Teorema de La Base Media.

Figura 28: Menor Media en un Tri´angulo Rect´angulo. 3. Uniendo el punto A  con el punto P   se forman los tri´angulos congruentes AM P  y ACP  (por L A L) ya que M A = P C , AP  = AP , ∠MAP  = ∠AP C   (por ´angulos alternos internos entre las paralelas  M A  y  P C ). Luego,  M P  =  AC , entonces  N P  = 21 M P  = 21 AC . Adem´as, ∠P AC  = ∠MP A, de donde  M P  AC  o que  M N  AC .

 

 

Corolario: Menor mediana de un tri´ angulo rect´ angulo.  En todo tri´angulo rect´angulo, la mediana relativa a la hipotenusa es la mitad de la longitud de la hipotenusa y es la menor de las tres medianas del tri´angulo. Demostraci´ on:  En la figura 28, BM  es la mediana relativa a la hipotenusa AC  del ABC , probaremos que BM  = AC   ; (con lo cual se tendr´a que BM  =  AM  =  M C ). Si por M  se traza 2 una paralela al lado AB , que corte al lado BC  en N , entonces N  es el punto medio de BC  y el ∠M N C  = 90 , los tri´angulos BN M  y CN M  son congruentes por el criterio L-A-L, luego MB =  M C  =  AM . Probar que BM  es la menor mediana (Ejercicio).

 

18

Ejercicios.

1. (*) En la figura adjunta, ABC  es un tri´angulo equil´atero y  CDEF  es un cuadrado. Se construye un punto G tal que CF  = CG   y adem´as ∠CF G = 15 . Probar que ∠AGC  = ∠BDC .

2. Dado un tri´angulo equil´atero ABC , se construye un tri´angulo equil´atero DEF   cuyos v´ertices est´an sobre los lados del ABC , tal como muestra la figura 29. Demuestre que los tri´angulos  ADF ,  B ED ,  C F E  son todos congruentes entre si.

 

Figura 29 3.   ABCD  es un cuadrado,  E ,  F ,  G  y  H  son puntos sobre los lados  AB ,  B C ,  C D,  DA, respectivamente, tal que  EF GH   tambi´en es cuadrado. Demuestre que los tri´angulos AEH , BF E , CGF ,  DHG  son todos congruentes entre si. Figura 30.

Figura 30

19

4.   ABCDE  y   FGHIJ   son pent´ agonos regulares (Vease figura 31). Demuestre que los tri´angulos  AF J ,  B GF , CHG, DI H , EJ I  son todos congruentes entre si.

Figura 31 7

5. Si AB

  CD y AB  =  C D   entonces, AD  =  BC  y AD  BC  .

6. Demuestre que dos tri´ angulos desplazados son congruentes.  Sugencia:   Utilice el problema anterior.

7. Demuestre que dos tri´angulos rotados son congruentes.

7

El cuadril´atero  ABCD  se denomina paralelogramo.

20

8. Demuestre que dos tri´angulos reflejados con respecto a un punto

8

son congruentes.

9. Demuestre que dos tri´ angulos reflejados con respecto a una recta son congruentes.

Importante: Las traslaciones, rotaciones y reflexiones no cambian el tama˜no ni la forma de un tri´angulo.

10. (*) En la figura adjunta, ABCD  un cuadrado y EF  Demuestre que que EF  =  GH .

⊥ GH .

11. Dos cuadrados  ABCD y   EHGF , ambos de lado l, est´an colocados en manera tal que un v´ertice de uno est´a en el centro del otro (como en la figura anexa). Demuestre que el ´area del cuadril´atero  EJBK  es y por ende no depende de la posici´on de J  (o K ).

8

l2

4

La reflexi´on con respecto a un punto es equivalente a una rotaci´on de 180

21

12. En un ABC  el AB , si  F C  = 9.

 

∠B

= 2∠C , la mediatriz del lado AC   corta en F   al lado BC . Hallar

13. (*) (Examen final de XVI Olimpiada mexicana de Matem´ atica) Los ´angulos de un tri´angulo ABC  est´an en progresi´on aritm´etica (∠B ∠A  = ∠C  ∠B =  θ ), D, E , y F   son los puntos medios de los lados BC , CA y AB , respectivamente. Llamamos H   al pie de la altura trazada desde C   (que cae entre B y F ) y G  a la intersecci´on entre DH  y EF . ¿Cu´anto vale ∠F GH ?



14. En la figura 32, AC  = 12 AF  = 4 y

∠BAF  =



30. Hallar BF  si AG  =  GC .

Figura 32 15. En la figura 33, AG  =  GC , el

∠AF G  =

20 . Hallar el

∠F AC ,

si  AC  = 2 BF .

Figura 33 16. (*) Sea  ABCD  un cuadrado. Se construyen tri´angulos equil´ ateros  ADP  y  AB Q  como se muestra en la figura 34. Sea  M  la intersecci´on de  C Q  con  AD  y  N  la intersecci´on de  C P  con  AB . Demuestre que CMN  es un tri´angulo equil´atero.

22

Figura 34 Problemas de Refuerzo.

17. En la figura 35, ABC , CDE  y EF A  son tri´angulos is´ osceles, con el BDF   es equil´ atero. ∠EF A  = 120 . Probar que el

∠ABC  = ∠CDE  =

 

Figura 35 18. (*) ABC  es un tri´angulo is´ osceles con ∠ABC  = tal que ∠ABD  = 10  . Demuestre que AD  =  BC .

 

23

∠ACB

= 80  . D  es un punto en AC 

4.

Cuadril´ ateros: Clasificaci´ on y Propiedades.

´ CLASIFICACION. Los cuadril´ateros pueden clasificarse de acuerdo a sus diagonales de la siguiente forma:

Cuadril´atero Convexo:  Es un cuadril´atero con las dos diagonales en su interior. Cuadril´atero Entrante:  Es un cuadril´atero con una diagonal en el interior y otra en el exterior. Cuadril´atero Cruzado  Es un cuadril´atero con las diagonales en su exterior.9 Es muy frecuente que se considere que un cuadril´atero es convexo, a menos que se especifique lo contrario. Esto es as´ı porque muchos resultados son m´ as claros en un cuadril´atero convexo, sin embargo, es importante darse cuenta que existen teoremas que no se cumplen para cualquier tipo de cuadril´ateros, por ejemplo:

Teorema:  La suma de los a´ngulos internos de un cuadril´atero no cruzado es 360. La demostraci´o n de este resultado se basa en la disecci´on del cuadril´atero en dos tri´angulos cuyos ´angulos internos conforman los a´ngulos internos del cuadril´atero, sin embargo, estas condiciones no pueden lograrse en un cuadril´atero cruzado; de hecho, la suma de los ´angulos internos puede hacerse arbitrariamente peque˜na cuando el cuadril´atero es cruzado. Tambi´en hay otras clasificaciones de cuadril´ ateros de acuerdo a sus lados y ´angulos.

Cuadril´ atero Equi´ angulo:  un cuadril´atero (convexo) es equi´angulo si todos sus a´ngulos internos son iguales; dado el teorema anterior, los ´angulos son iguales a 90, por ello este cuadril´atero es llamado  rect´  angulo. Cuadril´ atero Equil´ atero:  un cuadril´atero (convexo) es equil´atero si todos sus lados son iguales. A este cuadir´atero tambi´en se le conoce como  rombo. Cuadrado: es un cuadril´atero que es equi´angulo y equil´atero. Paralelogramo:  es un cuadril´atero con los lados opuestos paralelos. Trapecio:  es un cuadril´atero con un par de lados opuestos paralelos. 10 9

Tanto los cuadril´ateros convexos como los entrantes son cuadril´ateros   simples , que son los cuadril´ateros cuyos lados no se cortan salvo en los extrenos; en contraposici´on, los cuadril´ateros cruzados no son simples. 10 Note que un paralelogramo es tambi´en un trapecio.

24

PARALELOGRAMOS Dado el paralogramo  ABCD , por propiedades de ´angulos entre paralelas es posible probar el siguiente resultado:

Teorema:   Los a´ngulos opuestos son iguales y los a´ngulos consecutivos son suplementarios: θ. ∠ABC  = ∠CDA  =  θ y ∠BC D  = ∠DAB  = 180



Por otra parte, por criterio ALA, i.e.

ABC  ≡ CDA; esto implica que  AB = C D  y  B C  = DA ,

Teorema:  Los lados opuestos de un paralogramos son iguales. A partir de esto, si  M  es la intersecci´on de  AC  con  BD , por criterio ALA, por lo que AM  =  C M  y BM  =  DM , i.e.

ABM  ≡ CDM ,

Teorema:  Las diagonales de un paralelogramo se bisecan. Adem´ as, se cumple un resultado sofisticado y muy importante:

Teorema: Ley del Paralelogramo. Si  ABCD  es un paralelogramo entonces el doble de la suma de los cuadrados de los lados es igual a la suma de los cuadrados de las diagonales, es decir 2 AB 2 + BC 2 =  AC 2 + BD 2





Demostraci´ on:  Aplicando la Ley del Coseno a AC 2 = DB 2 = 2

2

 ABC  y ABD  se tiene AB + BC  − AB · BC  cos θ AB + AD − AB · AD cos(180 − θ ) 2 AB + BC  − AB · BC  (cos θ  + cos(180 − θ ))



2

2

2

2

2

2



⇒ AC  + DB = y dado que cos θ  = − cos(180 − θ ) el resultado se sigue inmediatamente. ´ RECTANGULOS

En primer lugar, es importante notar que todo rect´angulo es paralelogramo (por ´angulos entre paralelas), por lo que todos los resultados probados anteriormente son heredados a todo rect´angulo; pero los rect´angulos tienen propiedades adicionales: ABD , por lo que AC  =  BD  y entonces Observe que por criterio LAL, ABC  Teorema:  Las diagonales de un paralelogramo son iguales; adem´as, el punto de intersecci´on de estas equidista de los cuatro v´ertices y por tanto es el centro de una circunferencia que pasa por todos los v´ertices.

 

 ≡ 

Por otra parte, observe que si se aplica la ley del paralelogramo a un rect´angulo se obtiene el Teorema de Pit´  agoras . 25

ROMBOS CDA, y por lo tanto ∠BAC  = ∠DAC  Dado un rombo  ABCD , por criterio LLL, ABC  y ∠BC A = ∠DAC , lo cual implica BC  AD y AB CD, i.e., todo rombo   ABCD   es un paralelogramo. Adem´ as, por las mismas congruencias se tiene

  

 ≡ 



Teorema:  Las diagonales de un rombo cumplen ser una mediatriz de la otra. Teorema:  Las diagonales de un rombo bisecan a los ´angulos interiores del rombo; esto implica que el punto de corte de las diagonales equidista de los cuatro lados del rombo y es el centro de una circunferencia tangente a estos.

TRAPECIOS Dado el trapecio  ABCD  (con AB CD), se construyen los puntos medios de BC  y  DA,  M  y N , respectivamente. Si el cuadril´atero  MN AB  se rota con centro en  M  y ´ angulo 180 se genera       un cuadril´atero MN  A C ; observe que N D = N  A y N D N  A , por lo que DN N  A  es un paralelogramo y

 

 

N N  = DA 

2M N  = DC  + CA 2M N  = DC  + AB AB  + CD M N  = 2



El segmento MN  es llamado  base media  del trapecio, y por lo reci´ en demostrado se tiene

Teorema:  La base media de un trapecio es igual a la semisuma de las bases. angulo es Por otra parte, hay ciertos trapecios que reciben nombres particulares; el  trapecio rect´  aquel que las bases son perpendiculares a alguno de los otros lados; y por otra parte, el  trapecio is´  osceles  es aquel que los lados (distintos de las bases) tienen igual longitud. 11

Ejercicios 1. Dado el trapecio   ABCD con AB CD, demuestre que la bisectriz interior del paralela a la bisectriz exterior del ∠D.



∠A

es

2. A un rombo   ABCD   se le construyen exteriormente los cuadrados   ABEF  y   BCGH . Demuestre que ABD  = EB H .

 



3. (*) Sea  ABCD  un paralelogramo. Se construyen tri´angulos equil´ ateros exteriores y ADQ. Demuestre que el BP Q  es equil´atero.

 

 

CDP 

4. Demuestre que las bisectrices interiores de un paralelogramo forman un rect´angulo (¿qu´e sucede si el paralelogramo es adem´as rombo?). 11

Los trapecios is´osceles son muy importantes cuando se estudian los ´angulos en la circunferencia; resulta que un trapecio es is´osceles si y s´olo si los cuatro v´ertices se ubican sobre una misma circunferencia.

26

5. Demuestr Demuestree que las bisectrice bisectricess exteriore exterioress de un paralelogra paralelogramo mo forman un rect´ rect´angulo. angulo. 6. Sea   ABCD   un paralelogramo. La bisectriz interna del ∠C DA  corta a BA en M , y la bisectriz interna del ∠BAD  corta a  C D en N . Demuestre que  ADNM  es   es un rombo. 7. Demuestre que si por el punto de intersecci´on o n de las diagonales de un rombo se trazan perpendiculares a los lados del rombo, entonces los puntos de intersecci´on de dichas perpendiculares con los lados del rombo forman un rect´angulo. angulo. 8. Demues Demuestre tre que las bisect bisectric rices es de los ´angulos angulos definidos por las diagonales de un rombo, cortan a los lados del rombo en cuatro puntos que forman un cuadrado. 9. En un ABC  sea G la intersecci´ intersecci´on on de las medianas BB   y C C  . Sean B  , C  las reflexiones de  G  respectivas a los puntos B  y  C  .



a) Demuestr Demuestree que  AGCB  y  AGBC   son paralelogramos. b) A partir de lo anterior anterior,, demuestre demuestre que BC B  C   tambi´en en es paralel par alelogr ogramo amo.. c) Demuestr Demuestree que A   pertenece a la recta AG, y concluya que las tres medianas de un tri´angulo angulo concurren en el punto G, llamado el  centroide  del ABC . d) Demuest Demuestre re que C G  = 2GC  ; relaciones similares se cumplen para las otras dos medianas.

 

10.  Teorema de Varignon:  Dado un cuadril´atero atero  ABCD  (no necesariamente convexo), se  B C ,  C D , construyen los puntos medios  L ,  M ,  N ,  O ,  P ,  Q , de los segmentos de recta  AB ,  BC  DA, BD ,  AC , respectivamente. Figura 36. a) Demuestr Demuestree que  LMNO ,  LPN  LPN Q,  OPMQ, son paralelogr paralelogramos. amos. b) Demuest Demuestre re que LN , OM , P Q  concurren en un punto, llamado el  centroide   del cuadril´atero atero  ABCD .  B D; resultados similares se c) Demuestre que el per´ per´ımetro de  LMNO  es igual a AC  +  +  BD cumplen cumplen para los otros paralelogramos. paralelogramos.

Figura 36: Teorema de Varignon

27

11. Sea   ABCD   un paralelogramo tal que existe un punto E  sobre el lado AB  que cumple ∠C ED  = 90. Sean M  y N   los pies de las perpendiculares trazadas desde A y B   hacia DE  y  C E , respectivamente. Demuestre que AC , BD y M N  concurren. 12. (*) (H´ector ector Alberti) Sea   ABCD   un cuadrado. Se construyen los tri´angulos angulos equil´ateros ateros         BDA BD A ,  AC B ,  B DC  y AC ACD D . Demuestre que el A B C  D  es tambi´en en un cuadrad cua drado. o. 13. (*) (II Olimpiada Olimpiada Matem´ Matem´ atica del Cono Sur) En la figura 37  ABCD y  AECF  son paraatica lelogramos. Demuestre que  BEDF   es paralelogramo.

Figura 37 Problema Problemas s de Refuerzo. efuerzo.

14. (*)  ABCD  es un cuadril´atero atero convexo y O  es un punto en su interior. Sean P , Q, R, S , los puntos medios de los lados AB , BC , C D, DA, respectivamente. Por P   se traza una paralela a OR , por Q  se traza una paralela a OS , por R  se traza una paralela a OP , y por S  se traza una paralela a OQ . Demuestre que estas cuatro rectas concurren. 15. (*) Un trapecio trapecio is´ osceles osceles tiene diagonales perpendiculares perpendiculares y su ´area area es 2010, determine su altura. 16. (*) (IX Competencia de Clubes Cabri, Segunda Ronda) Sea   ABCDEF   un hex´agono agono regular cuyo centro es O. Se construyen los cuadrados  FSOP  y  ORCQ. Demuestre que APQB y  SEDR  son rect´angulos. angulos. Figura 38. 17. (*) Sobre los lados del ABC  se trazan exteriormente los cuadrados  ABPQ,  CARS  y BCTU . Luego se trazan los paralelogramos AQA R, C SC  T  y  B U B  P .

 

a ) Sean A , B  , C   los centros de los cuadrados  BCTU ,  CARS ,  ABPQ, respectivamente. Demuestre que estos centros est´an an sobre los lados del A B  C  .

Demuestree que AA ,  B B  , C C   concurren. b ) Demuestr

28

 

Figura 38 18. (*) Se dibujan cuadrados cuadrados exteriores exteriores a los lados de un paralelogramo, paralelogramo, demuestr demuestree que: cuadril´ l´ atero determinado por los centros de esos cuadrados es un cuadrado. atero a ) El cuadri b ) Las diagonales diagonales de ese cuadrado cuadrado son concurren concurrentes tes con las del paralelogramo paralelogramo..

19. (*) Dado un ABC , se constr construy uyen en exteri exteriorm ormen ente te los tri´ tri´angulos angulos rect´ rectangulo a´ngulo is´ osceles osceles ACP  y BC Q, con AC  y BC   como hipotenusas. Si M   es el punto medio de AB , demuestre que el M P Q  tambi´  tamb i´en en es un tri´angulo angulo rect´angulo angulo is´ osceles. osceles.



 



 

29

5.

´ Angulos en la Circunferencia.

LA CIRCUNFERENCIA Y SUS ELEMENTOS Una   circunferencia  es el lugar geom´etrico de puntos que equidistan de un punto dado, llamado el centro  de la circunferencia; la distancia de cada punto de la circunferencia al centro es el  radio. Por otra parte, todos los puntos que est´an a una distancia del centro menor o igual al radio forman el  c´ırculo ; estos puntos quedan “al interior” o sobre la circunferencia. Si  A  y  B  son dos puntos de una circunferencia, el segmento de recta  AB  define una  cuerda ; en particular, si el centro de la circunferencia pertenece a la cuerda, ´esta es llamada di´  ametro. Es importante mencionar que para cada punto de la circunferencia existe exactamente un punto diametralmente opuesto. En la figura 39, se tiene una circunferencia de centro O   y radio r = OA = OB = OA ; AB y AA   son cuerdas, pero AA  es tambi´en di´ametro, i.e, A  es diametralmente opuesto a A y viceversa. Observe que por la desigualdad triangular aplicada al tri´angulo is´ osceles AOB

 

Figura 39

AB < AO  + BO

= r + r = AA Si  A  es un punto fijo, esta desigualdad es v´alida para cualquier punto  B  sobre la circunferencia (excepto cuando B = A  lo cual implica AB = AA ). Esto quiere decir que el di´ametro es la mayor de todas las cuerdas. A las porciones de circunferencia que quedan entre dos puntos ubicados en la circunferencia, se les llama  arcos de circunferencia ; note que dos puntos sobre una circunferencia definen dos arcos de circunferencia. Tambi´en, si un ´angulo tiene v´ertice sobre el centro de la circunferencia y est´a formado por dos radios, ser´a llamado ´  angulo central ; de nuevo, ∠AOB  hace referencia a dos ´angulos, cuya suma es 360, y  subtienden  respectivamente a uno de los arcos AB . Finalmente, si un ´angulo tiene el v´ertice sobre la circunferencia y est´a formado por dos cuerdas, ser´a llamaangulo inscrito ; en la figura anterior, ∠AA B  es un ´angulo inscrito que subtiende al arco  AB . do ´ 

30

Teorema:  El ´angulo central es el doble del ´angulo inscrito que subtiende el mismo arco. Demostraci´ on:  Considere la figura 40, se demostrar´a que ∠AOB  = 2∠AP B  en los tres casos mostrados. En la circunferencia de la izquierda, sea P   el punto diametralmente opuesto a P ; observe que AP O y BP O  son tri´angulos is´ osceles, y por el teorema del ´angulo externo se tiene

 

 

∠AOB

= = = = =





∠AOP   + ∠BOP 

(∠AP O + ∠OAP ) + ( ∠BP O + ∠OBP ) 2∠AP O + 2 ∠BP O 2 (∠AP O + ∠BP O) 2∠AP B

Figura 40 El caso de la circunferencia del medio es m´as sencillo y se deja como ejercicio para el lector. Para la circunferencia de la derecha, el trabajo es an´alogo y s´olo cambia en un peque˜no arreglo algebraico ∠AOB

= = = = =



∠BOP 

− ∠AOP 

(∠BP O + ∠OBP ) (∠AP O + ∠OAP ) 2∠BP O 2∠AP O 2 (∠BP O ∠AP O) 2∠AP B

− −



Corolario:  Todos los ´angulos inscritos que subtienden el mismo arco son iguales (Ver figura 41). En particular, los ´angulos internos son iguales a 90 si subtienden a una semicircunferencia. Demostraci´ on: Todos los ´angulos mostrados en la figura 41 son iguales a la mitad del ∠AOB , y por tanto, son iguales entre s´ı. En particular, si AB  fuera un di´ametro, ∠AOB  = 180   y por tanto ∠AP B = 90  . 12 Hay un par de ´angulos m´ as que son importantes: Si un punto  P  es interno a la circunferencia, el ´angulo de v´ertice P  formado por dos cuerdas que pasan por P  se llama a´ngulo  interior . De 12

Observe que en cualquier tri´angulo rect´angulo, el punto medio de la hipotenusa equidista de los tres v´ertices.

31

Figura 41 forma similar, si  P  es exterior y dos cuerdas de la circunferencia (al prolongarse) pasan por  P , el ´angulo con v´ertice  P  es llamado a´ngulo  exterior . Dejamos como ejercicio demostrar el siguiente teorema: ormulas Teorema:   Los ´angulos interior y exterior mostrados en la figura 42 cumplen las f´ siguientes: ∠AQC 

=

∠AP C 

=

∠BOD  + ∠AOC 

2 ∠BOD

− ∠AOC  2

Figura 42

´ CUADRILATEROS C´ICLICOS Ahora suponga que sobre una circunferencia se ubican cuatro puntos A, B , C , D, como se muestra en la figura 43. Al cuadril´atero   ABCD   se le llama   cuadril´  atero c´ıclico o  conc´ıclico. Observe que α  β  +  = 180◦ . ∠ABC  + ∠CDA  = 2 2 32

Figura 43 Y an´alogamente ∠DAB + ∠BC D  = 180  . Esto significa que si  ABCD  es un cuadril´atero c´ıclico y convexo, entonces los ´angulos opuestos son suplementarios. Tambi´en, es posible demostrar por contradicci´on el rec´ıproco de este resultado: si suponemos que  ABCD  es tal que ∠B + ∠D  = 180 pero no es c´ıclico, se define el punto D  como la otra intersecci´o n de AD  con el circunc´ırculo del ABC , y como   ABCD    es c´ıclico (por construcci´on) entonces ∠B  + ∠D  = 180, luego,  on CD  C D  (rectas paralelas que se cortan en C ). ∠D = ∠D , lo cual implica la contradicci´ As´ı, se ha demostrado el siguiente teorema:

 

 

Teorema:  El cuadril´atero convexo  ABCD  es un cuadril´atero c´ıclico si y s´olo si ∠A

+ ∠C   = 180◦  =

∠B  + ∠D

Tambi´en, otro criterio muy u ´ til y cuya demostraci´on tambi´en se basa en el corolario anterior es

Teorema:   El cuadril´atero convexo   ABCD   es un cuadril´atero c´ıclico si y s´o lo si se cumple alguna de las siguientes igualdades = ∠BC A = ∠BAC  = ∠CAD =

∠ABD

∠ACD ∠BDA ∠BDC  ∠CBD

Es importante recalcar que NO todo cuadril´atero puede ser inscrito en una circunferencia; por ejemplo, un paralelogramo no ser´a c´ıclico a menos que sea rect´angulo.

RECTAS Y CIRCUNFERENCIAS TANGENTES A UNA CIRCUNFERENCIA Dada una circunferencia, una recta puede ser  tangente  o   secante   a la circunferencia, dependiendo si la corta en  uno  o  dos  puntos, respectivamente; en cualquier otro caso, se dice que la 33

recta no corta a la circunferencia. 13 Sea l   una recta secante a la circunferencia que corta a la circunferencia en A y B (A = B ); como el AOB  es is´osceles, ∠OAB <  90. Rec´ıprocamente, si por A  se traza una recta l tal que uno de los ´angulos que forma con OA  es menor que 90, se puede construir un punto B sobre l  tal que ∠OAB = ∠ABO < 90 y A = B  (basta proyectar O  sobre l  y luego reflejar A con respecto a este punto, el resultante es el punto B ); entonces el AOB  es is´osceles, por lo que OA  =  r =  OB , i.e. B  pertenece a la circunferencia y por tanto l  corta a la circunferencia en dos puntos distintos. As´ı

 

 

 

 

Teorema:  Una recta  l  corta a una circunferencia de centro  O  en dos puntos distintos  A  y  B si y s´olo si un a´ngulo entre  l y OA  es agudo. Corolario: Si l  es una recta tangente en A  a una circunferencia de centro O, ninguno de los ´angulos entre l y  OA  puede ser agudo, y por tanto l OA.



A partir de este resultado se prueban otros resultados muy conocidos y ´utiles, que dejamos de ejercicios para el lector.

Teorema:  Dado un punto  P  externo a una circunferencia de centro  O , si  P A  y  P B  son segmentos tangentes a la circunferencia en  A  y  B , respectivamente, entonces el cuadril´atero  P AOB es c´ıclico y bis´osceles. Corolario:  Dado un punto P  externo a una circunferencia de centro O , la circunferencia de di´ ametro  P O  corta a la circunferencia dada en dos puntos  A  y  B  tales que  P A  y  P B  son rectas tangentes. Definici´ on: El ´angulo semi-inscrito  en una circunferencia es aquel que se forma con una cuerda y la recta tangente en alguno de los extremos de la cuerda. Teorema:  La media del ´angulo semi-inscrito definido por la cuerda AB  es igual a la medida de un ´angulo inscrito que subtiende al arco  AB . Demostraci´ on:   Considere la figura 44. Como  APBO  es c´ıclico, entonces adem´ as, como P O  es la mediatiz de AB , ∠P OB  = ∠P OA, por lo que ∠P AB  =

∠AOB

2

∠P AB

=

∠P OB ;

= ∠AQB

Por otra parte, dada una circunferencia, otra circunferencia puede ser   secante  o  tangente  a la primera, dependiendo si la corta en  uno  o  dos  puntos, respectivamente; en cualquier otro caso se dice que las circunferencias no se cortan. 14 13

Cuando la recta es tangente a la circunferencia puede considerarse como un caso muy peculiar en el cual los “dos” puntos de corte coinciden. 14 Tambi´en ac´a puede considerarse a las circunferencias tangentes como un caso especial de circunferencias secantes en el cual los puntos de corte coinciden.

34

Figura 44 Adem´ as, dos circunferencias pueden posicionarse una dentro de la otra, y claramente, la circunferencia de radio mayor es la  externa   mientras que otra es la  interna ; particularmente, si las circunferencias tienen el mismo centro se llaman   conc´entricas . Finalmente, combinando estas definciones se tienen las circunferencias  tangentes exteriormente  y las  tangentes interiormente .

Teorema:  Dadas dos circunferencias de centros  O 1 y  O 2  que se cortan en dos puntos distintos A  y  B , se cumple que O1 O2 AB .

 ⊥

Teorema:  Si dos circunferencias de centros O1 y O2  son tangentes en A, se cumple que  O 1 ,  A y O2  est´an alineados. Teorema: a) Dos circunferencias, una dentro de la otra, no tienen rectas tangentes en com´un. b) Dos circunferencias tangentes interiormente tienen una recta tangente com´un. c) Dos circunferencias secantes (en dos puntos distintos) tienen dos rectas tangentes en com´un. d) Dos circunferencias tangentes exteriormente tienen tres rectas tangentes en com´un. e) Dos circunferencias no secantes y tal que ninguna contiene a la otra, tienen cuatro rectas tangentes en com´ un.

Ejercicios

1. Si el ∠M P Q  = 20, determine el valor del adjunta.

∠QON  en

35

la figura

2. Dado un a´ngulo inscrito  BAC , y su ´angulo central  BOC , se sabe que 180 . Calcular el ∠OBC .

∠BAC  + ∠BOC  =

3. En la figura 45,  BCDO  es un rombo. Determine el valor del ´angulo  θ  y la medida de las diagonales de BCDO  si el radio de la circunferencia mide 6.

Figura 45 4. Un cuadril´ atero c´ıclico  ABCD  satisface

∠ABC  = 2 ∠CDA  =  θ .

5. En la figura 46, P R  es una tangente com´un. Calcule el valor del

Calcule θ. ∠P QR.

Figura 46

6. En la figura adjunta, el 150 . Calcule el ∠AGB .

∠AF E   =

100 y el

∠BC D

=

OA y m OB . Si P   es el punto de 7. Dado un ´angulo ∠AOB , se trazan dos rectas l corte de l y m, demuestre que A, B , O, P  se ubican sobre una misma circunferencia.

 ⊥

36

 ⊥

8. Las bisectrices BP  y CQ del entonces P QI  es is´osceles.

 

 ABC   se cortan en I . Demuestre que si ∠BAC  = 60

9. En la figura 47 se ha tomado un punto C   sobre la circunferencia; AC  y BC  cortan a la segunda circunferencia en  D y  E  respectivamente. Probar que OC  DE .

 ⊥

Figura 47 10. (*) Dada la figura 48, demuestre que AB

  AB.

Figura 48 11. En la figura 49 CR  es una recta tangente en C , demuestre que  AB

Figura 49

37

  CR.

12. Dos circunferencias Γ1  y Γ2  son tangentes (interior o exteriormente) en  P  (Ver figura 50). Dos rectas que pasan por P   cortan a Γ1 y Γ2 en A y C , y en B y D, respectivamente. Demuestre que AB CD.

 

Figura 50

13. (*) Dos circunferencias de centros  O 1 y  O 2  son tangentes (interna o externamente) en un punto P ; por este punto se traza una recta que corta nuevamente a la circunferencias en A  y B , respectivamente. Demuestre que AO1 BO 2 .

 

14. Dos circunferencias son tangentes externamente en el punto A. Una tangente exterior com´ un toca a una circunferencia en B  y a la otra en C . Demostrar que ∠BAC  = 90  . 15. En la figura 51, DE  es tangente en D, y C  es el punto medio del arco  AD . Encuentre el valor del ´angulo seminscrito ADE .

Figura 51

38

16. Determine el valor del ∠DCF , sabiendo  BE   es tangente en el punto  D  a la circunferencia de centro O. Ver Figura 52.

Figura 52 17. Si el

∠AEB  =

30,

∠ADE   =

20 y

∠ACE  =

35, calcule el

∠AF B .

V´ease figura 53.

Figura 53 18. Dada una circunferencia de di´ametro  BC , se toma un punto  P  en la prolongaci´on de  BC , y se traza la tangente  AP . Si  AP  =  AB y O  es el centro de la circunferencia, demuestre que el AOC  es equil´atero.

 

19. (*) Dadas dos circunferencias una fuera de la otra, demuestre que las tangentes comunes externas forman segmentos iguales; an´alogamente, las tangentes comunes internas forman segmentos iguales. 20. (*)  Teorema de Pithot.   Demuestre que en todo cuadril´atero inscribible, la suma de lados opuestos es igual. 21. (*)  Teorema de Steiner.  En todo cuadril´atero exinscrito a una circunferencia, la diferencia de las longitudes de lados opuestos es igual. 39

22. Demuestre que las mediatrices de un cuadril´ atero son concurrentes si y s´olo si es c´ıclico. 23. Demuestre que el cuadril´atero convexo   ABCD   es inscribible si y s´olo si los inc´ırculos respectivos del ABC  y CDA  son tangentes.

 

 

24. Demuestre que las bisectrices internas de un cuadril´atero son concurrentes si y s´olo si es inscribible. 25. Demuestre que todo rombo es inscribible. 26. En la figura 54,  AB  es una cuerda y por  D  se traza una recta tangente a la circunferencia paralela a AB . Demuestre que  C D  es bisectriz del ∠ACB .

Figura 54 27. Determine las medidas de

∠ACB

y

∠ACO  de

la figura 55.

Figura 55

28. Cuatro cilindros de di´ametro 1 est´ an pegados apretadamente por una cuerda muy fina, como en la figura adjunta. Demostrar que la cuerda tine longitud 4 +  π . Demostrar tambi´ en que el ´area somπ breada entre los cilindros es 1 4 .



29. En la figura 56,  ABCD  es un trapecio is´osceles con  AB CD  y  DA  =  BC   = 2; tomando DA y BC  como di´ ametros, se construyen dos circunferencias tangentes. Si DC  = 3AB , calcule el ´area del trapecio.

 

40

Figura 56 30. La figura 57 est´a formada por un paralelogramo y dos circunferencia tangentes entre s´ı y tangentes a tres lados del paralelogramo. Sabiendo que el radio de las mismas mide la cuarta parte del lado menor del paralelogramo, calcule la raz´on entre el lado mayor del paralelogramo y el radio de las circunferencias.

Figura 57 31. En la figura 58,  ABCDEF  es un hex´agono regular y las circunferencias de centro en los v´ertices son tangentes dos a dos. Si las circunferencias sobre los v´ertices B , D, F  son iguales, demuestre que las circunferencias restantes son iguales.

Figura 58 32. Alrededor de una circunferencia se construyen diez circunferencias tangentes a la original y tangentes entre s´ı (V´ease figura 59). Demuestre que la suma de las ´areas de las diez circunferencias es el doble del ´area de la circunferencia mayor. 41

Figura 59 33. (*)  Teorema de Miquel:  Dado un ABC , sean X , Y  , Z   puntos sobre AB , BC , CA, respectivamente. Demuestre que los circunc´ırculos de AXZ , BY X , CZY  tienen un punto en com´ un M .

 







34. (X OMCC - P2, Aar´on) Sea  ABCD  un cuadril´atero conc´ıclico con di´ametro AC , y sea O   el centro de su circunferencia. Se construyen los paralelogramos   DAOE  y   BCOF . Demuestre que si E  y F   est´an sobre la circunferencia entonces  ABCD  es rect´angulo. 35. (*) Sea ABC  un tri´angulo, y sean L y N  las intersecciones de la bisectriz del ´angulo A con el lado BC  y el circunc´ırculo de ABC  respectivamente. Construimos la intersecci´on M   del circunc´ırculo de ABL   con el segmento AC . Prueba que los tri´ angulos BM N  y BM C  tienen la misma ´area. 36. (*) Sea AB  el di´ametro de una semicircunferencia. Se colocan los puntos M  y K   sobre la semicircunferencia y sobre  AB , respectivamente.15 Sea  P  el centro de la circunferencia que pasa por A, K  y M ; sea Q  el centro de la circunferencia que pasa por B , K  y M . Demuestre que  M P K Q  es conc´ıclico. 37. (*) Las circunferencias Γ 1 y Γ2  se cortan en los puntos A y B . Por el punto A  se traza una recta que corta nuevamente a las circunferencias Γ 1 y Γ2   en los puntos C  y D, respectivamente. Por los puntos C  y D   se trazan tangentes a las circunferencias, las cuales se cortan en el punto  M . Demuestra que  MCBD  es c´ıclico. 38. (*) El ABC   cumple que ∠A = 90 y AB = AC . Se toma un punto E   del segmento AB , se construye interiormente un tri´angulo equil´ atero AEF . EF    corta BC  en I , y se construye exteriormente un tri´angulo equil´atero BI J . Encuentre ∠EJ B .

 

39. (*) En la figura 60, se sabe que ∠AO1 B ∠AO2 B  = 70◦  y adem´as la tangente  E B  forma el tri´angulo is´ osceles  AB E , con AB  =  AE . Encuentre ∠EBC .



15

M  y  K  son distintos de  A  y  B .

42

Figura 60 40. (*) Dos circunferencias Γ1  y Γ2  se cortan en  A  y  B . Una recta por  A  corta a Γ 1  y Γ2  en  C  y  D , respectivamente, y la paralela a  C D  por  B  corta Γ1  y Γ2  en  E  y  F , respectivamente. EAF . Demuestre que CDB

 

 ≡ 

41. (*)   La Recta de Simson-Wallace.   Sean X , Y  y Z   los pies de las alturas trazadas desde un punto P   en el circunc´ırculo del ABC  hacia AB ,  B C  y CA, respectivamente. Demuestre que X ,  Y  y Z   est´an alineados.

 

42. (*) Sea  P  un punto exterior al cuadrado  ABCD  tal que ∠AP C  = 90◦ , Q  es la intersecci´on de  AB y  P C , y  R  el pie de la perpendicular por  Q  a  C A. Demuestre que  P ,  R  y  D  est´an alineados. 43. En la figura 61,  ABCD  es un trapecio rect´angulo tal que la circunferencia de di´ametro AB   (y centro O) es tangente a CD . Demostrar que O  pertenece a la circunferencia de di´ametro CD  y que esta circunferencia es tangente a BA .

Figura 61 44. El ABC  es rect´angulo en C , la circunferencia de centro  O  es tangente a cada uno de los lados del ABC  en los puntos P ,  Q  y  R  (como se muestra en la figura 62), y se cumple que AP  = 20 y  B P  = 6. Calcule OP .



 

43

Figura 62 45. En la figura 63 se muestran tres semicircunferencias, una de di´ ametro AB  (de centro O y radio r), otra de di´ametro AO  y la ´ultima de di´ametro OB . Determine la raz´on entre el radio de la circunferencia tangente a estas tres semicircunferencias y r.

Figura 63 46. El segmento AB  es di´ametro de un semic´ırculo con centro en O. Un c´ırculo con centro en P  es tangente a AB en O  y tambi´en al semic´ırculo. Otro c´ırculo con centro en Q es tangente a AB , al semic´ırculo y al c´ırculo de centro en P . Si AB  = 2, ¿cu´al es el radio del c´ırculo con centro en Q?

Figura 64 Problemas de Refuerzo.

47. Los v´ertices A y B  de un tri´angulo equil´atero ABC  est´an sobre una circunferencia de radio 1 y el v´ertice  C  est´a en el interior de la circunferencia. Un punto  D  (distinto de  B ) que esta en la circunferencia es tal que AD  =  AB . La recta  DC  corta por segunda vez a la circunferencia en E . Encuentre la longitud del segmento  C E . Ver figura 65.

 

48. (*) (OIM 2002, P-4) En un tri´ angulo escaleno  ABC   se traza la bisectriz interior  BD , con D  sobre AC . Sean E  y F  puntos sobre la recta BD  tales que ( AE  CF ) BD , y sea M  el punto sobre el lado  B C  tal que DM  BC . Demuestre que ∠EM D  = ∠DM F .

 ⊥

44

 

 ⊥

Figura 65 49. (*) (OMCC 2003, P-2) Sea S  una circunferencia y  AB  un di´ametro de ella. Sea  t  la recta tangente a  S  en  B  y considere dos puntos  C  y  D en  t  tales que  B  este entre  C  y  D . Sean E  y F  las intersecciones de S  con AC  y AD  y sean G y H  las intersecciones de S  con CF  y  DE . Demuestre que AH  =  AG . 50. (*) (The 59th Romanian Mathematical Olympiad District Round) Considere un cuadrado ABCD  y un punto  E   sobre el lado  AB . La diagonal AC  corta al segmento  DE  en el punto P . La perpendicular por  P  a DE  corta al lado BC  en F . Probar que  E F  =  AE  + CF . 51. (*)   Teorema de Arqu´ımedes:   En la figura 66, la regi´on delimitada por tres semicircunferencias mutuamente tangentes, es conocida como  cuchilla de zapatero o arbelos  . ´  Demostrar que las circunferencias sombreadas son congruentes.

Figura 66: Teorema de Arqu´ımedes.

45

6.

Teorema de Thales y su rec´ıproco. Semejanza de Tri´ angulos.

Introducci´ on. Definici´ on 1. Raz´on: se llama raz´on, al cociente de dos cantidades, expresadas en la misma magnitud, por ejemplo ab . 2. Proporci´ on: se llama proporci´on a la igualdad de dos razones. Por ejemplo ab = dc , 16 a los t´erminos a  y  d  se les llama extremos y los t´erminos b  y  c  se les llama medios, al t´ermino d  se le llama cuarta proporcional entre a,  b y c  en este orden. Propiedades de las proporciones: 1.

a c =  si y s´olo si  a c  =  b d. b d

2.

a c b d a b =  si y s´olo si = o = . b d a c c d

3.

a c a b c d =  si y s´olo si = . b d b d

4.

a c a+b c+d =  si y s´olo si = . b d a b c d

·

·

± −

± −

Paralelismo y proporcionalidad. Definici´ on 1. Un punto  P 

 ∈ AB  divide al segmento AB  en una raz´on dada r, si

PA PB

=  r .

Figura 67 2. Sean  AB y  C D  dos segmentos, y sean P  AB y  Q CD, decimos que  P  y  Q  dividen a QA A AB y CD  en segementos proporcionales si PP B = QB .

 ∈



Figura 68 16

En algunos textos de geometr´ıa se utiliza la notaci´on de proporci´on as´ı a  :  b  ::  c  :  d  que se lee “ a  es a  b  como

c  es a  d ”.

46

Teorema de Thales.  Si tres paralelas cortan a dos secantes entonces los segmentos que determinan en ellas son proporcionales. 17 Antes de demostrar el Teorema de Thales, se enunciar´an dos teoremas que a pesar de su apa´ rente sencillez es de mucha utilidad en problemas que involucran Areas y Proporcionalidad.

Lema 1.  Sea  AB

  CD. Demuestre que: ( ABC ) = ( ABD ).

Lema 2.  Sea  P  un punto sobre el lado AB  (o su prolongaci´on) del

 ABC . Pruebe que:

AP  (AP C ) = PB (P BC )

. A continuaci´on se enuncian los pasos a seguir en la demostraci´on del teorema de Thales. Demostraci´ on.  Sean  AA  ,  BB   y  C C   rectas paralelas que cortan a dos secantes en los puntos A, A , B , B  , C , C   respectivamente (ver figura 69).

Figura 69: Teorema de Thales Pruebe que: AB (ABB  ) 1. = BC  (BC B  ) A B  (A B  B ) 2.   = . B C  (B  C  B )

3. (ABB  ) = (A B  B ) y (BC B  ) = ( B  C  B ). Con ayuda de las igualdades demostradas concluya que: AB A B  =  . BC  B C 

Observaci´ on Importante:  Utilice las propiedades de las proporciones para demostrar las equivalencias siguientes (interpr´etelas geom´etricamente): AB A B  =   BC  B C 

AC  A C  AC  A C  ⇔ AB =   ⇔ =   AB BC  B C 

 

17

 

El teorema de Thales puede enunciarse de manera general como sigue: Si tres o m´as paralelas cortan a dos o m´ as secantes entonces los segmentos que determinan en ellas son proporcionales.

47

Corolario (Teorema de Thales en el tri´angulo). Toda recta paralela a un lado de un tri´angulo y que corte a los otros dos lados, divide a estos lados en segmentos proporcionales. Rec´ıproco del Teorema de Thales.  Si tres rectas cortan a dos secantes en segmentos proporcionales y dos de estas rectas son paralelas entonces las tres rectas son paralelas. Demostraci´ on. Sean  AA  ,  BB   y  C C   rectas que cortan a dos secantes en los puntos  A ,  A  ,  B , AB A B      B , C , C   respectivamente, tales que AA CC  y =   . Por el punto B   tracemos una BC  B C  recta paralela a AA , la cual interseca a A C   en el punto D  (ver figura 70). Entonces, por el AB A D A B  A D Teorema de Thales se tiene que: =  . De donde, B  C  = DC  , as´ı por las propiedades BC  DC  A C  A C  de las proporciones   = , por lo que  B  C   =  B  D + DC   =  DC   y por tanto  B  D  = 0,  B C  DC  o equivalentemente  B   =  D  y por lo tanto,  B B  AA .

 

 

Figura 70: Rec´ıproco del Teorema de Thales

Corolario (Rec´ıproco del Teorema de Thales en el tri´ angulo.)  Si una recta intercepta dos lados de un tri´angulo en segmentos proporcionales entonces la recta es paralela al tercer lado del tri´angulo. Tri´ angulos semejantes.  Decimos que el cual denotamos as´ı AB C  A B  C  , si:

 ∼

 ABC  es semejante al ABC   (Ver figura 71), lo

AB AC  BC  = = A B  A C  B  C 

y ∠BAC  = ∠B

 A C  , ∠ABC  = ∠A B  C  , ∠ACB  =

∠A

 C  B  .

En los tres teoremas que se muestran a continuaci´on (los cuales son una consecuencia directa del Teorema de Thales) se establecen las condiciones m´ınimas para demostrar que dos tri´ angulos son semejantes, a los cuales denominaremos:  Criterios de Semejanza de Tri´  angulos .

Primer criterio de semejanza de tri´ angulos: Angulo-Angulo A-A.  Si dos a´ngulos de un tri´angulo son congruentes con dos ´angulos de otro tri´angulo, entonces los dos tri´angulos son semejantes. 48

Figura 71: Definici´ on de Semajanza de Tri´angulos.

Demostraci´ on.  Supongamos que en el ABC  y A B  C   se tiene que ∠ABC  = ∠A B  C  y ∠ACB = ∠A C  B  , entonces ∠BAC  = ∠B  A C    (Por la suma de ´angulos internos en un tri´angulo). Sea D AB y E  AC   tales que AD = A B  y AE  = A C  , dado que ∠DAE  = ∠BAC  =    ADE  = A B  C  , por consiguiente ∠ADE  = ∠A B  C  = ∠B A C  , se sigue por L-A-L que angulos correspondientes) y por el teorema de ∠ABC , de donde  DE  BC  (por ser iguales los ´ Thales AB AC  =

 

 ∈

 ∈

 

 

 

 

AD

AE 

y por consiguiente

AB AC  = (1) A B  A C  Sea F  BC  tal que DF  AC , entonces F C  =  DE  =  B  C   (porque  DECF   es paralelogramo y por ser ADE  = A B  C  ) y por el teorema de Thales

 ∈

 



 

BA BC  = DA F C 

o lo que es lo mismo

AB BC  = A B  B  C 

(2)

Luego, de (1) y (2) se tiene que: AB AC  BC  . = = A B  A C  B  C 

As´ı, se ha demostrado que los tres pares de ´angulos son congruentes y los tres pares de lados son proporcionales, por lo tanto, ABC  A B  C  .

 

 ∼ 

Segundo criterio de semejanza de tri´ angulos: L-A-L.  Si un ´angulo de un tri´angulo es congruente con otro ´angulo de otro tri´ angulo y los lados que comprenden al ´angulo en el primer tri´angulo son respectivamente proporcionales a los lados que comprende al ´angulo en el segundo tri´angulo, entonces los dos tri´angulos son semejantes. AB AC  Demostraci´ on.  Suponga que el ∠BAC  = ∠B  A C   y que   =   . Considere los punAB A C  tos D y E , como en la demostraci´on del teorema anterior. Entonces por el criterio L-A-L, 49

ADE  = ABC , de lo cual se deduce que

∠ADE 

=

∠A

 B  C  . Por otra parte tene-

AB AC  = , y al aplicar el rec´ıproco del teorema de Thales, se puede afirmar que AD AE  DE  BC , de lo cual a su vez se deduce que ∠ADE  = ∠ABC , por ´ angulos correspondientes entre paralelas. Finalmente por transitividad se concluye que ∠ABC  = ∠A B  C  . Por lo tanto, ABC  A B  C   (Por el criterio A-A.)

mos que:

 



 ∼ 

Tercer criterio de semejanza de tri´angulos: L-L-L.  Si los tres lados de un tri´angulo son respectivamente proporcionales a los tres lados de otro tri´angulo, entonces los dos tri´angulos son semejantes. AB

AC 

BC 

Demostraci´ on.  Por hip´otesis se tiene que:   =   =   y como antes sean D y E  AB A C  B C  puntos sobre AB y AC   respectivamente tales que AD = A B  y AE  = A C  . Entonces por el rec´ıproco del teorema de Thales se tiene que DE  BC   y por consiguiente el ∠ABC  = ABC  ADE  (por el criterio A-A). Por ende ∠ADE  y el ∠ACB = ∠AED , de donde AB BC  BC  BC  = , luego por transitividad =   , de donde DE  = B  C  . En consecuencia AD DE  DE  B C  ADE  = A B  C    (por el criterio L-L-L), de lo cual se sigue que ∠A B  C  = ∠ADE  y          ∠A C  B = ∠AED , y por transitividad ∠A B C  = ∠ABC  y ∠A C  B = ∠ACB   =. Por lo ABC  (Por el criterio A-A.) tanto, A B  C 

 ∼

 





 

 ∼ 

Ejercicios 1. Sean AB y CD  las bases del trapecio  ABCD , cuyas diagonales son perpendiculares. Si se sabe que AD  = 13,  AE  = 12 y  C E   = 4 encuentre las longitudes de  C D y AB . 2. En la figura 72, el ABC  es equil´atero, sus lados tienen longitud 3 y P A  es paralela a BC . Si P Q  =  QR  =  RS , encontrar la longitud de CS .

 

Figura 72 3. Sea  ABCD  un trapecio de bases BC  y AD , sus diagonales se cortan en E . Si BE  = 3, ED  = 4 y CE   = 2, determine la medida de AE . 4. Las bases de un trapecio miden 3 y 5, y si su altura mide 4. Encontrar la distancia desde el punto de corte de las diagonales hasta la base mayor.

50

5. En la figura adjunta, el ABC  es rect´angulo en A y el ADB es rect´angulo en  D . El punto  E  es el punto de intersecci´on de los segmentos AD y BC . Si AC   = 15, AD  = 16 y BD  = 12, calcule el ´area del ABE .





 

6. El

ABC  es rect´angulo en B . Se dibuja un rect´angulo BEDF  con D  sobre la hipotenusa,

E  y  F   sobre BC  y AB , respectivamente. Si  AB  = 1, demuestre que

BC  BE 

= 1−1DE .

7. Consid´erese los puntos A, B , C  y D   tales que A y B   est´an sobre el segmento OC  y OD   respectivamente, donde O   es el centro de la circunferencia de radio r   (Ver figura DOC   y que CD = 73). Si OA OC  = r2 = OB OD , demuestre que el AOB



2

r OA OB

·

·

·

 



AB .18

Figura 73 8. Sobre la circunferencia de centro O, se trazan los di´ametros AB  y CD tales que AB CD. Sea P  un punto sobre el arco CBD y Q  el punto de intersecci´on de las cuerdas AP  y CD. Si DO  = 1, demuestre que AP  AQ  = 2.

 ⊥

·

9. Un segmento de recta AB   es divido por los puntos interiores K  y L   de manera que AL2 =  AK  AB . Sea  P  un punto exterior al segmento  AB  tal que  AP  =  AL . Pruebe que ∠KP L  = ∠LP B . Figura 74.

·

Figura 74 18

La medida del segmento  C D  se denomina  Distancia Inversa .

51

10. En la figura 75,  AB y  AC  son tangentes a la circunferencia, y  C E  di´ametro. Probar que BE  BO = AB  C E .

 ⊥ BD , siendo  B D un

Figura 75 11. Demostrar que

1 AX 

+

1 BY 

=

1

 si se cumple que AX  BY 

 

AZ 

 CZ . (Ver figura 76.)

Figura 76 12. En la figura 77, el ABC    es rect´angulo. Se construyen exteriormente los cuadrados ABEF  y  BCPQ. Demostrar que BM  =  BN .



Figura 77: .

52

13. Sean O, P  y R  los centros de las tres circunferencias. Si OR = r y Q  es la intersecci´on de  P O  con la circunferencia de centro R, demuestre que OP  OQ  =  r 2 . Ver figura 78.

·

Figura 78 14. Si en un tri´angulo rect´angulo se traza la altura correspondiente a la hipotenusa, entonces: angulos que resultan, son semejantes entre si y semejantes al a ) Los dos nuevos tri´ tri´angulo original. b ) La altura es media proporcional

19

entre los segmentos que ella determina sobre la

hipotenusa. on del cateto c ) Cada cateto es media proporcional entre la hipotenusa y la proyecci´ sobre la hipotenusa. d ) Demuestre el teorema de Pit´agoras.

15. Si dos tri´angulos tienen sus lados respectivamente paralelos o respectivamente perpendiculares, entonces los dos tri´angulos son semejantes. 16. Las alturas, las bisectrices y las medianas hom´ ologas de dos tri´angulos semejantes est´an en la misma raz´on que sus lados hom´ologos. 17. Sean ABC  y A B  C   dos tri´angulos semejantes con AAB = BBC  = C CAA = k . Demuestre B C  que: la raz´on entre los per´ımetros de los tri´angulos es k  y que la raz´on entre sus ´areas es k2. 











18.  Teorema de Menelao.  Dado el ABC , sea  P  un punto sobre la recta  AB ,  Q  un punto sobre la recta  B C , R  un punto sobre la recta CA. Si los puntos  P , Q, R  est´an alineados



entonces

AP  BQ CR = 1. P B QC  RA

Para demostrar este teorema, sea W  un punto sobre la recta P QR  tal que BW 

  AC :

a ) Demuestre que los tri´ angulos AP R  y  B P W  son semejantes. Si  b  es una magnitud tal que ab = cb , entonces decimos que  b  es media proporcional entre  a  y  c ,o de manera equivalente:  b  es media proporcional entre  a  y  c  si y solo si  b 2 =  a c. 19

·

53

Figura 79: Teorema de Menelao. angulos CQR  y BQW  son semejantes. b ) Demuestre que los tri´ c ) De los literales a ) y b ) deduzca que

AP  BQ CR = 1. P B QC  RA

19.  Teorema de Ceva.  Dado el ABC , sea  P  un punto sobre el recta  AB ,  Q  un punto sobre la recta BC  y  R  un punto sobre la recta CA. Si las rectas AQ, CP , BR  concurren, entonces



AP  BQ CR = 1. P B QC  RA

Para demostrar este teorema, sean  W  y  V  los puntos de intersecci´on de la recta que pasa por B  paralela a AC , con las rectas CP  y  AQ , respectivamente. a ) Demuestre que

 AP C  ∼ BP W  y que AQC  ∼ V QB. b ) Demuestre que BW P  ∼ RCP   y que BV P  ∼ RAP . c ) Utilice los literales a ) y b ) para probar que

AP  BQ CR = 1. P B QC  RA

20. Si dos cuerdas se interceptan en el interior de una circunferencia entonces el producto de las medidas de los segmentos determinados por el punto de intersecci´on en una de las cuerdas es igual al producto de las medidas de los segmentos determinados en la otra cuerda. 21. Si dos segmentos se interceptan en un punto que esta en el interior de los dos segmentos y el producto de las medidas de los segmentos determinados por el punto de intersecci´on en el primer segmento es igual al producto de las medidas de los segmentos determinados por el punto en el segundo segmento,entonces los extremos de los segmentos est´an sobre una circunferencia. 22. Si desde un punto P   exterior a una circunferencia se trazan dos semirrectas secantes que cortan a la circunferencia en los puntos A, B y C , D   respectivamente, entonces P A P B =  P C  P D.

·

·

54

23. Si desde un punto P  se trazan dos semirrectas con los puntos  A ,  B  sobre una y los puntos C , D  sobre la otra, tales que P A P B  =  P C  P D, entonces los puntos A, B , C , D  est´ an sobre una circunferencia.

·

·

24. Si desde un punto exterior a una circunferencia se trazan dos semirrectas, una tangente y la otra secante, entonces el segmento entre el punto y el punto de tangencia es media proporcional entre los segmentos determinados entre el punto exterior y los puntos de intersecci´on de la secante con la circunferencia. 20 25. Si  P  es un punto sobre el mismo plano que una circunferencia de centro  O  y radio r, y  d es la distancia del punto P  al centro  O  de la circunferencia, demuestre que: a ) Si P   est´ a en el interior de la circunferencia, entonces la potencia de P  es r2

2

−d . b ) Si P   est´ a en el exterior de la circunferencia, entonces la potencia de  P  es d − r . 2

2

a sobre de la circunferencia, entonces la potencia de  P  es cero. c ) Si P   est´

26. (*) (IV OMCC, P-4) Sea  ABC   un tri´angulo, D  el punto medio de  BC ,  E  un punto sobre el segmento AC   tal que BE  = 2AD y F   el punto de intersecci´o n de AD con BE . Si angulos del F EA . ∠CAD  = 60 , encuentre la medida de los ´

 

27. (*) Sea  ABCD  es un trapecio con  AD BC .  M  y  N  son los puntos medios de  C D  y  BC , M  respectivamente, y  P  el punto com´un de las rectas  AM  y  DN . Si PAP  = 41 , demuestre que ABCD  es paralelogramo.



28. Dado el ABC  se construye un cuadrado  PQRS  con  P  en  AB ,  Q  en  AC ,  R  y  S  en  BC . Sea  H  el pie de la altura desde A  hacia  B C . Demuestre que:



a )

1 PQ

=

1 AH 

+

1 BC 

b ) (ABC ) = 2(PQRS ) si y s´ olo si  AH  =  BC .

29. Sea P   un punto en el interior del ABC . Se trazan por P  las paralelas a los lados del tri´angulo, que queda dividido en tres tri´angulos y tres paralelogramos. Si las ´areas de los tres tri´angulos de la subdivisi´ on son, en alg´ un orden, 9, 16 y 25, hallar el ´area del ABC .

 



Problemas de Refuerzo.

30. (*) En la figura anexa, BC  = CD = DE  = EA = x y ∠AEB = 90  . Demuestre que ∠ABC  + ∠ACD  + ∠ADE  = 90  .

20

Los problemas anteriores nos permite establecer la siguiente definici´on de   Potencia de un punto con respecto a una circunferencia:  La potencia de un punto  P  con respecto a una circunferencia de centro  O  y radio  r  es el producto  P A P B , donde  A  y B  son los puntos de intersecci´on de la circunferencia con una recta que pasa por  P .

·

55

31. Las tres circunferencias de la figura 80 tienen el mismo radio  r , sus centros son colineales y la circunferencia de centro O2  es tangente a las otras dos. Por  A  se traza una tangente a la circunferencia de centro O3 . Obtenga el valor del segmento  B C  en funci´on de  r .

Figura 80 32. Sea ABCD  un rombo, con  AC  = 6 y  BD  = 8. Se construyen exteriormente los cuadrados ADEF  y  CDHG, cuyos centros son O1 y O2 , respectivamente (Vea figura 81). Calcular la medida del segmento O1 O2 .

Figura 81 33. Sea  ABCD  un cuadrado con P  y Q  sobre AB y BC  tales que BP  =  B Q. Sea H  el pie de la perpendicular de B a  P C . Demuestre que  DHQ  = 90 . 34. Dado un paralelogramo   ABCD , se trazan dos circunferencias tangentes externamente en P , y tales que la primera es tangente internamente al ∠ABC  y la otra es tangente internamente al ∠CDA, como en la figura 82. Demuestre que  B , P  y D  est´an alineados. 56

Figura 82 35. En un ABC  el ∠CAB  = 120. Encuentre la medida de la bisectriz interna del en funci´on de los lados adyacentes.

 

∠CAB

36. El ABC  tiene lados de 13, 14 y 15 unidades. El A B  C   est´a dentro del ABC  con lados paralelos a los de ´este y a 2 unidades de distancia de los lados del mismo. Calcule (ABC ) (A B  C  ).

 

 

 



37. (*) (Asi´atico Pac´ıfica) Sea ABC   un tri´angulo y D  el pie de la altura con respecto a A. Sean E  y F  puntos en una recta que pasa por D   (distintos de D) tales que AE  CE  y AF  BF . Sean M  y N  los puntos medios de BC  y EF , respectivamente. Demuestre que AN  N M .

 ⊥

 ⊥  ⊥

57

7.

Puntos y Rectas Notables del Tri´ angulo.

MEDIANAS Definici´ on:  En un tri´angulo, una   mediana  es el segmento de recta que une un v´ertice con el punto medio del lado opuesto. Teorema:  Las tres medianas de un tri´angulo concurren en un punto llamado el   Centroide 21 del tri´angulo y usualmente es denotado por G. Adem´as, las medianas de cortan mutuamente en raz´on 2:1. Demostraci´ on:  Dado el ABC  sean A , B  , C  , los puntos medios de BC , CA, AB , respectivamente. Defina G  como la intersecci´o n de BB  con CC  . Por el teorema de la base media, B  C  BC  y 2B  C  = BC ; observe que BC G B  C  G, con raz´ on de semejanza 2, por lo que

 

 

 

 

GB GC  = =2 GB  GC 

An´ alogamente, si G∗  =  AA 

∩ BB   se cumple G∗ B G∗ A = ∗  =2 G∗ B  G A

As´ı, G  y G∗  dividen al segmento BB   en dos segmentos cuya raz´on es 2:1, por lo que G  =  G ∗ , lo cual implica que AA , BB  , CC   concurren y GA GB GC  = = =2 GA GB  GC 

MEDIATRICES Definici´ on: Dado un segmento  AB , la  mediatriz  del segmento es el lugar geom´etrico de puntos que equidistan de  A  y  B , i.e., un punto  P   est´a sobre la mediatriz de  AB  si y s´olo si  P A  =  P B . Teorema:  La mediatriz de AB  es una recta l  perpendicular a AB   y que pasa por su punto medio. AB . En primer lugar Demostraci´ on: Sea M  el punto medio de AB , y l  pasa por M  y l se probar´a que todos los puntos de l  satisfacen la definici´on de mediatriz: Por definici´o n de punto medio M A  =  M B . por lo que claramente M  pertenece a la mediatriz de AB ; sea P  un punto de l  distinto de M , por criterio LAL, P M A P M B  por lo que P A  =  P B . Ahora, cabe preguntarse si existe alg´un punto fuera de l   que tambi´en cumpla la definici´on: suponga P   tal que P  A = P  B , esto implica que P  AB   es is´ osceles, y entonces ∠P  AB = ∠P  BA ; P  BM  , lo cual implisi M   es la proyecci´on de P   sobre AB , por criterio ALA P  AM  ca que  M  A  =  M  B , es decir que  M   =  M , y esto obliga a que P   est´e sobre  l  (ya que  P  M   =  l ).

 ⊥

   

≡

 

 ≡ 

Teorema: Las  mediatrices  de un ABC  concurren en un punto que equidista de los v´ertices del tri´angulo, llamado el  Circuncentro del ABC 

 

21

 

Tambi´en conocido como Geocentro, Centro de Gravedad, Baricentro, o m´as formalmente Equibaricentro.

58

Figura 83: Concurrencia de Mediatrices, Circuncentro y Circunc´ırculo. Usualmente, el circuncentro es denotado por  O , y  R  representa la distancia del circuncentro a los v´ertices R  =  OA  =  OB  =  OC  A esta distancia se le llama  Circunradio del ABC . As´ı, O  es el centro de una circunferencia que pasa por A,  B , C , cuyo radio es R, llamada el  Circunc´ırculo del ABC .22

 

 

Demostraci´ on:  Sea  O  la intersecci´on de las mediatrices de  AB y  BC , por el teorema anterior, como O  pertenece a la mediatriz de AB  se cumple OA  =  OB , y como tambi´en pertenece a la mediatriz de BC , OB = OC ; entonces OC  = OA, y utilizando de nuevo el teorema anterior, O  debe pertenecer a la mediatriz de  C A. As´ı, las tres mediatrices concurren en  O , y este punto equidista de los v´ertices del ABC .

 

Corolario:  Dado un tri´angulo, existe una circunferencia que pasa por los tres v´ertices (el circunc´ırculo); adem´as, esta circunferencia es ´unica. Una observaci´on importante es que la mediatriz del lado de un tri´angulo NO siempre pasa por el v´ertice opuesto; de hecho, esto s´olo se da si el tri´angulo es is´osceles.

ALTURAS La  altura  es un concepto que est´a intr´ınsecamente relacionado con la  distancia de un punto a  una recta ; la altura es la recta que debe trazarse para determinar esta distancia, i.e., es una recta que pasa por el punto y es perpendicular a la recta. A la intersecci´on entre la altura y la recta generalmente se le llama   pie de la altura , o tambi´en (m´as formal)   proyecci´  on  del punto sobre la recta. En particular, para tri´angulos, definiremos la altura de la siguiente forma:

Definici´ on:  Dado un tri´angulo, una   altura  es una recta que pasa por un v´ertice y es perpendicular al lado opuesto. 22

En ocasiones, denotaremos a esta circunferencia por Γ( ABC ).

59

Es importante observar que el pie de la altura NO siempre pertenece a un lado; de hecho, una altura puede estar “al interior” de un tri´angulo, coincidir con un lado, o estar completamente afuera de un tri´angulo.

Teorema: Las alturas  de un tri´angulo concurren en un punto, llamado el Ortocentro  del tri´angulo, usualmente denotado por  H .23 Demostraci´ on:   Dado el ABC , se construyen los puntos A1 , B1 , C 1 , tales que ABA 1 C , BC B1 A, CAC 1 B  son paralelogramos. Observe que el ABC  es el tri´angulo medial del A1 B1 C 1 , y que las alturas del ABC   son las mediatrices del A1 B1 C 1 ; como las mediatrices de un tri´angulo concurren (en este caso, las del A1 B1 C 1 ), las alturas del ABC  concurren.

 

 

  

 



 

La altura tambi´en puede escribirse en t´erminos de lugar geom´etrico:

Teorema:  La recta  l  es perpendicular a  AB  si y s´olo si  AL 2 LB 2 es constante. Es decir, que una recta perpendicular a  AB  es el lugar geom´etrico de los puntos  L  que satisfacen la condici´on anterior.



Demostraci´ on: sea  P  la intersecci´on de  l  con  AB , y  L  un punto arbitrario sobre  l ; por Pit´agoras se tiene AL2 LB 2 = AP 2 P B 2 , y el t´ ermino derecho de la igualdad es constante. La otra direcci´on de la implicaci´on se prueba por contradicci´on.





De esa definici´on tambi´ en puede fabricarsele una demostraci´ on del teorema anterior, sin embargo, no se aborda porque la prueba se basa en un resultado sofisticado llamadado   Teorema  de Steiner .24

BISECTRICES Definici´ on: La  bisectriz  de un ´angulo es una recta que “divide” al ´angulo en dos a´ngulos de igual magnitud. Teorema:  El lugar geom´etrico de puntos que equidistan de dos rectas dadas, generan un par de rectas perpendiculares llamadas  bisectriz interna  y  bisectriz externa  del ´angulo formado por las rectas. Demostraci´ on:  Suponga que las rectas se cortan en un punto  O ; sean  a ,  b  las rectas dadas, y P  un punto que equidista de ellas; si A  y B  son las proyecciones de P   sobre a y b, respectivamente, entonces P A  =  P B . Observe que por criterio LLL (utilizando Pit´agoras previamente), OAP  OBP , por lo que ∠P OA = ∠P OB , i.e., P   pertenece a la bisectriz del ∠AOB . Claramente aqu´ı se dan dos casos, recuerde que para definir el ´angulo entre  a  y  b  se utilizan u ´ nicamente semi-rectas, por lo que las rectas  a  y  b  definen cuatro ´angulos, que por parejas pueden



≡

23

angulo ´ ortico  del ABC . El tri´angulo formado por los pies de las alturas de un ABC  es llamado el  tri´ Sean l, m, n, tres rectas perpendiculares a los lados del AB , BC , CA del ABC , respectivamente. Sean L, M , N , puntos arbitrarios sobre l, m, n, respectivamente. Entonces las rectas l, m, n  concurren si y s´olo si AL2 + BM 2 + CN 2 =  N A2 + LB 2 + M C 2 .

 

24

60

 

 

ser opuestos por el v´ertice o suplementarios; de estos se escoge cualquiera de ellos como referencia, entonces, si ∠AOB  coincide con ´este o con el opuesto por el v´ertice, la recta  P O  es llamada bisectriz interna, y en caso contrario, bisectriz externa. As´ı, el lugar geom´etrico son dos rectas, y su perpendicularidad se basa en los pares de ´angulos que son suplementarios. Finalmente, si a b, el lugar geom´etrico es una recta paralela a a  y  b  que se ubica entre ellas a igual distancia de ambas (este es un caso extra˜ no de bisectriz interna, sin embargo, en ocasiones es u ´til tener esta convenci´on en mente; peor a´ un, la bisectriz externa es una recta ideal llamada  recta al infinito ).



Teorema: Las  bisectrices internas  de un ABC  concurren en un punto, llamado el  Incentro del ABC , usualmente denotado por  I . La distancia de  I  a los tres lados del tri´angulo es igual a un n´ umero r, llamado el   Inradio del ABC , y de aqu´ı que la circunferencia de centro I  y radio r  sea tangente a los lados del tri´angulo; dicha circunferencia es llamada el  Inc´ırculo del ABC .25

   





Figura 84: Concurrencia de Bisectrices Internas, Incentro e Inc´ırculo.

Demostraci´ on Sea I   la intersecci´on de las bisectrices internas de ∠A y ∠B   (obviamente, I  est´a en el interior del ABC ); como  I  pertenece a la bisectriz interna del ∠A, por el teorema anterior  dist(I,AB ) =  dist(I,AC ), y an´alogamente, como I  pertenece a la bisectriz interna del ∠B ,  dist(I,AB ) =  dist(I , C B ); entonces  dist(I,AC ) =  dist(I , C B ), y de nuevo por el teorema anterior y dado que I   est´a al interior del tri´angulo, I  pertenece a la bisectriz interna del ∠C . As´ı, las tres bisectrices internas concurren en un punto que equidista de los lados del tri´angulo.

 

Es importante notar que las intersecci´on de una bisectrices interna con el lado opueto del tri´angulo NO siempre coincide con el puntos de tangencia del inc´ırculo;26 de hecho, esto ocurre s´olamente si el tri´angulo es is´osceles.

Corolario:  Dado un tri´angulo, existe una circunferencia que es tangente interiormente a los tres lados (el inc´ırculo); adem´as, esta circunferencia es ´unica.27 25

En algunas ocasiones denotaremos al inc´ırculo por Λ( ABC ). En la figura, el ABC  es llamado  tri´  angulo tangencial  del DEF . 27 Existen 3 circunferencias m´as que son tangentes a los tres lados del tri´angulo, llamados  exc´ırculos ; estas circunferencias se ubican en el exterior del tri´angulo. 26

 

 

61

Ejercicios 1. Las ´areas de los seis tri´angulos AGB  , AGC  , BGA  , BGC  , CGA , CGB   son iguales e iguales a un 61  del ´area del tri´angulo ABC .

Figura 85 2. Los cuatro tri´angulos AB  C  , BC  A , CA B  , A B  C  ,28 son congruentes entre si y seme jantes al ABC  con raz´on de semejanza 21 .

 

3. El centroide del ABC  coincide con el centroide del tri´angulo medial A B  C  . Adem´as, estos dos tri´angulos tienen lados correspondientes paralelos (tri´angulos homot´eticos).





4. En la figura 86,  G  es el centroide. Si  GD  = 2 y el ´area sombreada vale 5, calcule  AD y el (ABC ).

Figura 86 5. Demostrar que las paralelas a los lados de un cada lado en tres partes iguales.

ABC , trazadas por el centroide  G  dividen

6.   ABCD  es un paralelogramo de centroide (baricentro) E , M  es el punto medio de AD , y F   es la intersecci´o n de AC  con BM . Si el ´area de   ABCD   es 1, calcule el ´area del cuadril´ atero  DEFM . 7. En el ABC , se traza la mediana AM . Demostrar que si BM  = AM , entonces el tri´angulo es rect´angulo en  A .



28

El

 A B C  es llamado el  tri´angulo medial  del  ABC . 





62

8. La suma de las distancias del centroide a los puntos medios de los lados de un tri´ angulo es 20. Calcule la suma de las medianas del tri´angulo. 9. La mediana tiene longitud menor que la semisuma de los lados adyacentes, es decir AA  < b + c, BB   < c + a, CC   < a + b. 10. Dado el ABC , sean D y E  puntos variables sobre los lados AB y  AC   respectivamente tales que BC  DE . Entonces, la mediana AA  puede definirse como el lugar geom´etrico de los puntos P  tales que P  CD BE .29

 

 

 ∈



11. Siempre es posible construir un tri´angulo XY Z  con las medianas AA , BB  , CC   de un ABC  dado. Adem´ as, los segmentos que unen el centroide del XY Z  con sus v´ertices son iguales a la mitad de los lados del ABC .



12. En el

 

 

 ABC , AB = BC   y la mediatriz de BC   interseca a la mediana BM  en L. Si

∠LCB  =

25, determine la medida del

13.  Ley del Seno.  Dado un

∠LAC .

 ABC , se cumple que sen∠A sen∠B sen∠C  1 = = = a b c 2R

14. Las reflexiones de H  con respecto a los lados del ABC  caen sobre el circunc´ırculo del mismo, es decir HH a  =  H a X   y an´alogo para los otros lados.

 

Figura 87 15. Las reflexiones de H  con respecto a los puntos medios de los lados del tri´angulo, caen sobre el circunc´ırculo del mismo. 29

Si  D  =  A  se define  P  =  A , y cuando  D  =  B   entonces  P  es punto medio de  B C .

63

16. Si O y H   son el circuncentro y el ortocentro de un ∠BAH  = ∠CAO . 17. La altura AH a  es bisectriz del

 ABC , respectivamente, entonces

∠H b H a H c .

18. Los circunc´ırculos de

 ABC , ABH , BC H , CAH  tienen igual radio.

19. La perpendicular trazada desde A  al lado H b H c  del tri´angulo o´rtico, pasa por el circuncentro del ABC .

 

atero ortoc´entrico, es decir que cada punto es el ortocentro 20. A,  B ,  C  y  H  forman un  cuadril´  del tri´angulo formado por los otros tres.

21. El ortocentro de un tri´ angulo est´ a al interior, sobre un v´ertice, o afuera del tri´angulo, si el tri´angulo es acut´angulo, rect´angulo, u obtus´ angulo, respectivamente. 22. El circuncentro del

 ABC  es el ortocentro del tri´angulo medial ABC . 23. Sea O  el circuncentro del  ABC . Si ∠AOC  = 100 y ∠OCB  = 30, determine la medida de los ´angulos del ABC . 24. Hallar los a´ngulos de un tri´angulo cuyo tri´angulo o´rtico tiene ´angulos de 20, 50 y 110. 25. Sea ABC  un tri´angulo obtus´ angulo de circuncentro O   y altura AD . Si ∠OCB  = 15, calcule el ∠DAB . 26. El ABC  de circuncentro O  y altura BD . Si ´angulos del tri´angulo  AB C .

 

∠DAB  =

35 y

∠OBD  =

∠OAB  =

25 y

10 encontrar los

27. En la figura 88, AB  es di´ametro de la circunferencia. Si X   es la intersecci´on de CG con AB , calcular el ∠CXB .

Figura 88

64

28. En el ABC , se trazan la altura AH   y la mediana BM . Demuestre que el is´osceles.

 

 M HC  es

 ABC  es rect´angulo en  C , ∠A = 75 y CH  es altura. Demuestre que CH  = AB . 4 30. Sea O   el circuncentro del   ABC  con ∠C   = 45 y sea D  el pie de la altura desde A. 29. Un

Calcule la medida del

∠ODC .

31. Dado el ABC  is´osceles con ∠A  = 90, sean  P  y  Q  son puntos dentro del tri´angulo tales que BP  =  AQ  y  AP  =  C Q. Si  B P  y CQ  se cortan en R, demostrar que  AR P Q.

 



32. Se ubican los puntos M  y K   sobre los lados BC  y CD  del cuadrado   ABCD , respectivamente, de modo que MC  =  K D . Sea P   la intersecci´on de M D y BK , demuestre que AP  M K .

 ⊥

33. Sean D, E , F  los puntos de tangencia ABC . Demuestre que se cumplen las per´ımetro del tri´angulo: AE  = BD = CD =



del inc´ırculo sobre los lados BC , CA, AB del siguientes relaciones, donde s   denota el semiAF  = s BF  = s CE  = s

−a −b −c

34. El ortocentro del

 ABC  es el incentro de su tri´angulo o´rtico.

35. Dado un ABC , su tri´angulo o´rtico y su tri´angulo tangencial tienen lados correspondientes paralelos (tri´angulos homot´eticos).

 

36. Las bisectrices exteriores de ∠B y ∠C , junto con la bisectriz interior de ∠A, concurren en un punto, llamado el   Excentro  con respecto al v´ertice  A , usualmente denotado por  I a . Este punto es equidistante a los lados del ABC , dicha distancia es el  Exradio  respecto a A, usualmente denotado por r a . As´ı, la circunferencia de centro  I a  y radio  r a  es tangente exteriormente a los lados del ABC , y es llamada el  Exc´ırculo  respecto a A.30



 

37. I  es ortocentro del

 I  I  I  . Adem´as se cumple: a b c

AX  = AZ  = s BX  = BY  = s CY  = CZ  = s

−b −c

38. En un ABC , la bisectriz exterior del ∠ABC  y la bisectriz exterior del ∠BC A  se cortan en D. La paralela a BC  por D  corta a AC  en L y a AB en M . Si LC  = 5 y MB = 7, hallar LM .



39. El

 ABC  es rect´angulo en  A . Si  I  es el incentro, calcular ∠BI C . 40. En un ABC , el ∠ABC  − ∠CAB  = 90. Sean D y E  los pies de las bisectrices interior y exterior del

30

∠BC A  respectivamente.

Demuestre que CD  =  C E .

An´ alogamente se definene los exc´ırculos con respecto a los otros v´ertices.

65

Figura 89 41. En el ABC ,  AB < AC ,  AD  es bisectriz, y  E  es un punto en  AB  tal que el ∠EDB  = 90. El punto  F  sobre AC  es tal que el ∠BE D  = ∠DEF . Demuestre que el ∠BAD  = ∠F DC .



42. En el ABC  se trazan las bisectrices interiores BD y  C E   tales que D  es el punto sobre AC ,  E  es el punto sobre  AB , 2∠BDE  = 3∠B y ∠CED  = 2∠B . Calcular los ´angulos del ABC .

 



43. Dado el ABC  con ∠A  = 90, sea D  el pie de la perpendicular desde A. Sean adem´as I  y  J  los incentros respectivos de ABD  y ACD. Demostrar que la bisectriz del ∠BAC  es perpendicular a IJ .

 





44. Un tri´angulo es is´osceles si cumple alguna de las siguientes condiciones: a) Dos medianas son iguales. b) Dos alturas son iguales. c) Dos bisectrices son iguales.31 45.   Teorema de la Bisectriz:  Dado el ABC , sean P  y  P   sobre BC . Se cumple que AP  y AP   son la bisectriz interna y la bisectriz externa del ∠A  si y s´olo si BP  BA BP  = =  P C  AC  P  C 

 

Sugerencia: Para demostrar la primera igualdad, trace CD gaci´on de AB . 31

  AP  con E  sobre la prolon-

Este caso es aparentemente tan sencillo como los anteriores, pero realmente es un resultado muy complicado y recibe el nombre de   Teorema de Steiner-Lehmus .

66

46. (*) De acuerdo con los datos de la gr´ afica 90, calcular el valor de  AB .

Figura 90 47. Dos circunferencias son tangentes internamente en P , y una cuerda AB  de la circunferencia de radio mayor es tangente en  Q  a la otra circunferencia. Ver figura 91. a) Demuestre que P Q  es bisectriz del ∠AP B . b) Llame A y B   a las otras intersecciones de P A y P B  con la circunferencia de radio menor y suponga que AB  = 15, P A  = 3 y P B   = 2; calcule  AQ  y  B Q.

Figura 91 48. (*) Sea  ABC   un tri´angulo tal que las medianas respectivas a  B y  C  son perpendiculares. Demuestre que se cumple la relaci´on. 5BC 2 =  C A2 + AB 2 . 49. (*)  Teorema de Poncelet:  Demuestre si 90  , entonces 2(r + R) =  b + c.

 ABC  es un tri´angulo rect´angulo con ∠A =

67

Problemas de Refuerzo.

50. (*) Sea ABCD  un paralelogramo. Q  es el punto medio de  AD, F   el pie de la perpendicular por B  sobre QC . Probar que AF  =  AB . 51. Dado el rombo   ABCD , se trazan las bisectrices internas de ∠DAC , ∠CAB , ∠BC A, ∠ACD , y cortan a DC , CB , BA , AD en P , Q, R, S , respectivamente. Demuestre que PQRS   es un rect´angulo. 52. (*) Sea  ABCD  un cuadril´atero tal que  AB =  C D. Las mediatrices de  AC  y  BD  se cortan en  P . Probar que ∠P AC  = ∠P CA  = ∠P BD  = ∠P DB . 53. (*)  AB C  es un tri´angulo y  P  un punto en su interior. Sean  A  ,  B  y  C   las reflexiones de P   sobre BC , CA y AB , respectivamente. D , E  y F   son los pies de las perpendiculares respectivos desde A, B y C   hacia B  C  , C  A y A B  . Probar que AD , BE  y CF  son concurrentes. 54. (*) (Arnoldo Aguilar) En la figura 92,  ABGH ,  BCFG y  CDEF   son cuadrados. Si I  es el centro de  ABGH  y J  =  DH  BG , demuestre que  I , J  y F   est´an alineados.



Figura 92 55. (*) (Arnoldo Aguilar) Sea ABC   un tri´angulo equil´atero. M  y  N  son los puntos medios de AB  y BC , respectivamente. Exteriormente al ABC  se construye un tri´angulo rect´ angulo ◦ is´osceles AP C , con ∠AP C  = 90 . Si  I  es la intersecci´on de AN  y MP , demuestre que CI  es la bisectriz de ∠ACM .



 

56. (*) En la figura adjunta, el ABC   es tal que ∠A = 90 y al es el radio de la circunferencia? ∠B  = 60 . ¿Cu´

 

57. (*) Dado el paralelogramo ABCD , sea M  el punto medio de AB , y N  la intersecci´on de CD  con la bisectriz interna del ∠ABC . Demuestre que M C  BN   si y s´olo si AN  es bisectriz del ∠DAB .



68

58. (*) En el ABC , se sabe que los v´ertices B , C , el circuncentro O  y el ortocentro H  del ABC  est´ an todos sobre una misma circunferencia.



 

a) Calcule el valor de ∠A. b) Demuestre que el incentro tambi´en pertenece al circunc´ırculo de  BCOH . 59. (*) Sea ABC  un tri´angulo de ortocentro H . Sean P  y Q  los pies de las perpendiculares desde H  a las bisectrices interior y exterior de  A , respectivamente. Si  M  es el punto medio de  B C , mostrar que  P ,  Q y M   est´an alineados. 60. (*) En un tri´angulo ABC , sea M  el punto medio de BC . Si se cumple que AB =  AC  y adem´as ∠M AC  + ∠ABC  = 90 ◦ , hallar ∠BAC .

 

61. (*) Sea  AB C  un tri´angulo y  U  un punto de su circunc´ırculo tal que AU  es bisectriz. Las mediatrices en AB y AC  cortan a AU  en X  y Y  . Sea T   la intersecci´on de BX  con CY  . Demostrar que AU  =  T B  + T C . 62. (*) (The 59th Romanian Mathematical Olympiad Final Round) Sea  ABCD  un rect´angulo de centro  O con  AB =  BC . La perpendicular en  O a BD  corta a las l´ıneas  AB y  B C  en los puntos  E  y  F , respectivamente. Sean  M  y  N  los puntos medios de los segmentos  C D y DA, respectivamente. Probar que las l´ıneas rectas F M  EN .

 

 ⊥

63. (*) Sea ABC  un tri´angulo rect´angulo, con A = 90◦ . Sea D  un punto en su interior tal que ∠DAC  = ∠DCA  = ∠DBC  =  α , y AC  =  BD . Determine el valor de α. 64. (*) Sea  AB C   un tri´angulo y  M  un punto tal que ∠M AB  = 10, 40 y ∠M CA  = 30. Probar que el ABC  es is´osceles.

∠M BA  =

20,

∠M AC  =

 

65. (*) En la figura 93,   ABCD y   PQRS   son cuadrados, ABP  BC Q CDR DAS  y los los radios de las cinco circunferencias son iguales a r. Si a  es el lado del cuadrado  ABCD , determine r  en funci´on de a.

 



Figura 93

69

≡

 ≡ 

 ≡

66. (*) Recta de Euler.  El centroide  G , el ortocentro  H  y el circuncentro  O  de un tri´angulo est´an alineados, y adem´as GH  = 2 GO. 67.   Circunferencia de los 9 puntos:32 Dado un ABC   de ortocentro H , se cumple que los puntos medios de los lados, los pies de las alturas, y los puntos medios de HA, HB , HC , se ubican sobre una misma circunferencia. Adem´as, el centro de esta circunferencia es el punto medio de  H O  y su radio es R2 , donde  O  y  R  son el circuncentro y el circunradio del tri´angulo. Para demostrar este resultado se sugiere seguir los siguientes pasos:

 

a) Si H a   es el pie de la altura trazada desde A, demuestre que la reflexi´o n de H  con respecto a H a  pertenece a circunc´ırculo del ABC . Resultados similares se cumplen para H b y  H c . b) Si A  es el punto medio de BC , demuestre que la reflexi´o n de H   con respecto a A

 

pertenece al circunc´ırculo del

 ABC .

c) De los resultados anteriores, observe que hay 9 puntos sobre el circunc´ırculo del ABC : los v´ ertices, las reflexiones de H   con respecto a los pies de las alturas, y las reflexiones de H  con respecto a los puntos medios de los lados; a partir de esto, concluya que los puntos medios de los segmentos que van de H   a estos 9 puntos, tambi´ en deben pertenecer en una misma circunferencia.



d) Concluya adem´ as que el centro de esta nueva circunferencia es el punto medio de  H O. Otro camino de soluci´on es el siguiente: a) Sea A B  C    el tri´angulo medial del ABC . Pruebe que ∠A B  C  = ∠BH a C  y concluya que H aA B  C   es un cuadril´atero c´ıclico; los mismo debe cumplirse para H b y  H c . b) Sea X  el punto medio de  HA. Demuestre que ∠B  A C  +∠B  XC   = 180 y concluya que XC  B  A  es un cuadril´ atero c´ıclico; lo mismo debe cumplirse para los puntos medios





de HB y  H C . c) De lo anterior, concluya que los pies de las alturas y los puntos medios de los segmentos que van desde H  hasta los v´ertices del ABC , se ubican sobre el circunc´ırculo del A B  C  . d) Si  N  es el circuncentro del A B  C  , demuestre que  N ,  O ,  G  forman la recta de Euler del A B  C   y utilice sus propiedades para probar que  N  es el punto medio de  H O.

 





 

68. El a´rea del

 ABC , denotada por [ABC ], cumple:   base × altura [ABC ] = 2

[ABC ] =

ab sen∠C 

2

=

 bc sen∠A

2

=

 ca sen∠B

2

=

abc 4R

[ABC ] = sr [ABC ] = 32

 

s(s

− a)(s − b)(s − c). (F´ormula de Her´on).

Tambi´en conocida como  Circunferencia de Feuerbach .

70

69. El circunradio, el inradio y los exradios de un tri´ angulo cumplen: 4R = ra + rb  + rc r [ABC ] = ra (s a) =  r b (s



r = ra = rb = rc =

b) =  r c (s

  −− − − −   − −   − − −   − − − (s

a)(s

b)(s

c)

− c)

s s(s b)(s c) s a s(s a)(s c) s b s(s

a)(s b) (s c)



70. Dado el

 ABC , sea  I  el incentro e I   el excentro respecto a A. a

a) Demuestre que  BICI a  es un cuadril´atero c´ıclico. b) Si M   es la intersecci´on de II a  con el circunc´ırculo del tri´angulo (M  =  A ), demuestre que dicho punto es el circuncentro de  BICI a. c) Sea  M   el punto diametralmente opuesto a  M  en el circunc´ırculo, y sea  P  la proyecci´on de I  sobre AB . Demuestre que M  CM  AIP .

 

 

  

d) Sea O  el circuncentro del tri´angulo Calcule la potencia de punto de I   con respecto ormula de Euler : al circunc´ırculo, y utilizando los resultados anteriores deduzca la F´  2 2 OI  =  R 2Rr .



e) A partir de la f´ormula de Euler demuestre que  R

71

≥ 2r.

8.

Soluci´ on a Problemas Selectos.

Teoremas Fundamentales del Tri´ angulo. 1. En la figura 94,  ABDE   es un cuadrado y BC D  es un tri´angulo is´ osceles con  B D  =  DC . Si ∠ABC  = 160, determinar la medida de ∠AEC .

Figura 94

Soluci´ on: ∠DBC  = ∠DCB  = 160 ∠ABD  = 70, de donde se obtiene que ∠BDC  = 40 y ∠EDC  = ∠EDB + ∠BDC   = 130. Como EDC   es is´osceles, entonces ∠DEC  = ∠DCE   = 25. Por lo tanto ∠AEC  = 90 ∠DEC   = 65.





 

2. Hallar la suma de los ´angulos α +  + θ + φ  en la figura 95.

Figura 95

Soluci´ on: ∠CAB = θ , ∠EDC  = α  por ser opuestos por el v´ertice. Como el externo en el BDF , se tiene ∠AF D  =  α + . Sumando los ´angulos internos del se tiene  α + φ + θ  +   = 180  .



∠AF D

AEF 

3. (XV Competencia de Clubes Cabri Primera Ronda) En la figura 96,   ABCD   es un rect´angulo tal que AB = 2BC . M   es el punto medio de AB   y los tri´angulos AM E  y M BF    son equil´ateros. Si P   es la intersecci´on de las rectas DE  y CF , encuentre los ´angulos del CDP .

 

72

Figura 96

Soluci´ on:  Note que AD = AE  = F B = BC  por lo que DAE  y BC F   son ambos is´osceles. Luego ∠DAE  = ∠CBF  = 90+60 = 150 lo que implica que ∠P DC  = ∠P CD  = 90 15 = 75 y luego ∠CP D  = 30.

 

 



4. Sea  AB C  un tri´angulo rect´angulo con ∠CAB  = 90 (Ver figura 97).  D  es un punto sobre la prolongaci´ on de BC  tal que BD =  BA. E  es un punto en el mismo semiplano que A respecto de BC , tal que CE  BC   y adem´as CE  = CA. Mostrar que A, D y E   est´an alineados.

 ⊥

Figura 97

Soluci´ on: Sea ∠CBA = 2θ ; el ABD   es is´osceles y ∠BAD  + ∠BDA = 2θ, por lo que ∠BAD = ∠BDA = θ. Como ∠CAB  = 90 entonces ∠ACB  + ∠ABC  = 90 y como CE   es perpendicular a BC   entonces ∠EC A  + ∠ACB   = 90; por lo tanto, ∠ABC  = EC A  es is´ osceles, ∠CEA = ∠CAE  = 90 θ . Luego, ∠EC A = 2θ . Con esto, como ı  E , A  y D  est´an alineados. ∠EAC  + ∠CAB  + ∠BAD  = 180 y as´

 

 



5. Dado un cuadrado ABCD , se construyen los tri´angulos equil´ ateros  ABP   (exteriormente) y ADQ  (interiormente). Probar que C ,  P  y Q  est´an alineados. Figura 98. 73

Soluci´ on:   Observe que ∠P AQ = ∠BAD   = 90 y P A = BA = DA = DQ , por lo que P AQ  es tri´angulo rect´angulo is´ osceles, y por tanto, ∠P QA  = 45. Por otra parte, CDQ   es is´ osceles ∠QDC  = 90 ∠ADQ   = 3 0 y QD = AD = CD , es decir, el con el ´angulo comprendido entre lados iguales de 30, por lo que ∠DQC    = 75. As´ı, an alineados. ∠P QA + ∠AQD + ∠DQC  = 180 y por lo tanto,  C , P  y Q  est´

 

 −



Figura 98 6. En la figura 99, AB = BC  = CD = DE  = EF  = F G = GA. Calcule la medida del ∠DAE . Referenciasfig20

Figura 99

Soluci´ on: Sea ∠DAE  = θ . Como los tri´angulos ABC  y AGF   son is´osceles, ∠ACB = ´ngulos externos de ABC  y AF G   se tiene ∠F BC  = ∠AF G = θ . Calculando los a GF E  y BC D  son is´ osceles, ∠GEF  = ∠BDC  = 2θ . Calculando ∠CGF  = 2θ . Como ´angulos externos de ADC  y AEF  se obtiene ∠EC D  = ∠DF E  = 3θ . Como CDE  y F ED  son is´osceles, ∠CED  = ∠F DE  = 3θ . Entonces, la suma de los ´angulos internos del AED da θ + 3 θ + 3 θ  = 180, de donde θ  = 180 . 7

   

  

 

   

 

 

7. (XXVIII Olimpiada Brasile˜ na de Matem´atica) En la figura 100, AB = AC , AM  = AN  y ∠CAM  = 30  , encuentre el valor del ∠BM N .

Soluci´ on: Como ABC  y AMN   son is´osceles, sean ∠ABC  = ∠ACB  =  α  y ∠AMN  = ormula del ´angulo externo se tiene ∠AN M . Por la f´





74

Figura 100

∠ACM  + ∠MAC 

=

α + 30

Esto implica que

∠BM N  =

∠AMB

=

= = =

∠AMN  + ∠BM N  ∠ANM  + ∠BM N 

(∠N BM  + ∠BM N ) + ∠BM N  α + 2∠BM N 

15 .

8. (Etapa semifinal Estatal de XXII Olimpiada Mexicana de Matem´aticas) En la figura 101 se muestra un hex´agono regular  ABCDEF  de lado 1. Los arcos del c´ırculo que est´an dibujados tienen centro en cada v´ertice del hex´agono y radio igual a la distancia al v´ertice opuesto. P , Q, R, S , T  y U   son los puntos de corte de estos arcos. ¿Cu´anto mide cada lado del hex´agono  P QRSTU ?

Figura 101

Soluci´ on:  El hex´agono  P QRSTU  es regular y con el mismo centro que  ABCDEF . Sea O  el centro de ambos (V´ease Figura 64). El lado buscado es igual a OP . Tenemos que CF  =  F P  =  P C   = 2 por ser radios de los arcos dibujados; entonces CF P  es equil´atero de lado 2 y  OP  es una altura de este tri´angulo que, por Pit´agoras, es igual a 3.

√ 

75

Congruencia de Tri´ angulos 1. En la figura 102,  ABC   es un tri´angulo equil´atero y  CDEF   es un cuadrado. Se construye un punto G  tal que  C F  =  C G  y adem´as ∠CF G  = 15  . Probar que ∠AGC  = ∠BDC .

Figura 102

Soluci´ on: ∠BC D  = 180 ∠ACB ∠DCF  = 30. Como GCF  es is´osceles, ∠CGF  = BC D ACG, ∠CF G  = 15 y ∠ACG = ∠CGF  + ∠CF G  = 30. Por criterio LAL, por lo tanto ∠BDC  = ∠AGC .





 

 

 ≡ 

2. (Cuaderno de Olimpiadas Mexicanas - Geometr´ıa) En la figura 103, ABCD  un cuadrado y EF  GH . Demuestre que que  E F  =  GH .

 ⊥

Figura 103

Soluci´ on:  Se construyen  EK  y  GM  con  K  sobre  C D  y  M  sobre  AD  tales que  EK  AD GHM , con  E F  =  GH . y GM  CD. Luego se demuestra que EF K 

 

 

 

 ≡ 

3. (Examen final de XVI Olimpiada mexicana de Matem´ atica) Los ´angulos de un tri´angulo ABC   est´ an en progresi´on aritm´etica (∠B ∠A = ∠C  ∠B = θ), D, E , y F   son los puntos medios de los lados BC , CA y AB , respectivamente. Llamamos H   al pie de la altura trazada desde C   (que cae entre B y F ) y G  a la intersecci´on entre DH  y EF . ¿Cu´anto vale ∠F GH ?

 −

Soluci´ on:  Note que

∠A + ∠B  + ∠C  =

 −

3∠A + 3 θ  = 180, lo cual implica que 76

∠A  +  θ

=

60 = ∠B . Entonces BC H  es un tri´angulo 30, 60, 90, y dado que  D  es punto medio de BC , el BDH   es equil´ atero. Luego, como  BC  EG , ∠F GH  = ∠BDH   = 60. Ver figura 104.



 

 

Figura 104 4. Sea   ABCD   un cuadrado. Se construyen tri´angulos equil´ ateros ADP  y ABQ   como se muestra en la figura 105. Sea M   la intersecci´o n de CQ con AD y N   la intersecci´o n de CP  con AB . Demuestre que CM N  es un tri´angulo equil´ atero.

Figura 105

Soluci´ on:   Note que P D = AD , porque AP D   es equil´atero, y AD = CD   porque ABCD  es cuadrado, por lo que  P D  =  C D , es decir, el CDP  es is´ osceles, con ∠CDP  = ∠CDA ∠P DA  = 30, entonces ∠DP C  = ∠DCP  = 75, y ∠BC N  = ∠BC D ∠DCP  = 15. An´alogamente, BC Q   es is´osceles con ´a ngulos 30, 75, 75, por lo que ∠MCN  = etrica con respecto a AC , ∠BC Q ∠BC N    = 60. Finalmente, como la figura es sim´ CM  = CN , entonces, el tri´angulo CM N  es equil´atero porque tiene dos lados iguales y un a´ngulo interno igual a 60.



−  −

5.



 



ABC  es un tri´angulo is´osceles con ∠ABC  = ∠ACB = 80 . D  es un punto en AC  tal

que

∠ABD  =

10 . Demuestre que AD  =  BC . Figura 106. 77

Soluci´ on: Se traza un punto  D   sobre  AC  tal que  AD   =  BC . Se construye exteriormente el tri´angulo equil´atero AEB . Luego, AE  = AB , D A = CB y ∠EAD  = ∠ABC  lo cual implica que EAD  ABC , de donde se deduce que el D EB   es is´ osceles     y ∠BE D = ∠BE A ∠D EA   = 40. Se sigue que ∠EB D   = 70 y como ∠D BA =  ∠ABE  = 10, resulta que  D   =  D  y por lo tanto  B C  =  AD   =  AD . ∠EB D

 





≡

 

Figura 106

78

Cuadril´ ateros. 1. Sea  ABCD  un paralelogramo. Se construyen tri´angulos equil´ ateros exteriores CDP  y ADQ, como se muestra en la figura 107. Demuestre que el BP Q  es equil´ atero.



 

 

Figura 107

Soluci´ on:   Observe que al hacer una rotaci´on de centro P  y ´a ngulo 60, el tri´angulo P BC   se transforma en el tri´angulo P QD  (observe los segmentos P C  y CB   tras esta transformaci´ on), mientras que al hacer una rotaci´on de centro  Q  y ´angulo 60, el tri´angulo P QD  se transforma en tri´angulo  B QA. Como la rotaci´ on mantiene las distancias,  P B = P Q  =  BQ, por lo que el tr´ıangulo BP Q  es equil´atero.33 2. (II Olimpiada Matem´ atica del Cono Sur) En la figura 108  ABCD y  AECF  son paralelogramos. Demuestre que  BEDF   es paralelogramo.

Figura 108 33

Una demostraci´on m´as rigurosa se basa en el c´alculo de los ´angulos ∠P CB = ∠P DQ  = en la utilizaci´on del criterio LAL para justificar P CB P DQ BAQ.

∠ABC  y

 

79

 ≡ 

≡

∠BAQ  =

120 +

Soluci´ o n 1: Sea M  el punto medio de AC . Las diagonales AC  y BD  se bisecan en M , mientras que las diagonales AC  y EF   tambi´en se bisecan en M , entonces BD y EF  se bisecan en M  por lo que  BEDF  es un paralelogramo. Soluci´ o n 2:  Como AD CB y AE  CF   entonces ∠DAE  = ∠BC F . Entonces, por propiedades de paralelogramos ∠BAE  = ∠BAD ∠EAD  = ∠BC D ∠BC F  = ∠F CD; DCF , y entonces  B E  = adem´as, AB  =  C D y  AE  =  C F . Por criterio LAL, BAE  DF . An´ alogamente se demuestre que ABF  CDE , lo cual implica BF  = DE . Como  BEDF   es un cuadril´atero con lados opuestos iguales, es un paralelogramo.

 



−   ≡

 

 ≡ 



3.   ABCD  es un cuadril´atero convexo y O   es un punto en su interior. Sean P , Q, R, S , los puntos medios de los lados AB , BC , CD, DA, respectivamente. Por P   se traza una paralela a OR , por Q  se traza una paralela a OS , por R  se traza una paralela a OP , y por S   se traza una paralela a OQ . Demuestre que estas cuatro rectas concurren.

Soluci´ on: Al tomar las rectas OP  y OR  y sus paralelas se forma el paralelogramo PORM , y al tomar las rectas  OQ  y OS  y sus paralelas se forma el paralelogramo  OQNS . Por el teorema de Varignon, sabemos que  P QRS   es un paralelogramo, y llamaremos  T  al punto de corte de sus diagonales. Observe que el punto de corte de las diagonales de  P ORM  es el punto medio de  P R, i.e.,  T ; an´alogamente, el punto de corte de las digonales de  OQN S  es el punto medio de SQ , i.e.,  T  nuevamente. As´ı, M  es la reflexi´on de O  con respecto a T , y de igual forma queda definido N , por lo que  M  =  N  y las cuatro rectas concurren. 4. (H´ector Alberti) Sea  ABCD  un cuadrado. Se construyen los tri´angulos equil´ ateros BDA , ACB  , BDC  y ACD    (V´ ease figura 109). Demuestre que el A B  C  D   es tambi´en un cuadrado.

Figura 109

Soluci´ on:   Como A B = A D, AB = AD, CB = CD, C  B = C  D, los puntos A , A, C ,  C   pertenecen a la mediatriz de  B D , y por tanto, est´ an alineados. An´alogamente, B  , B , D , D    est´ an alineados; por lo tanto A C  B  D . Por otra parte, si O   es el centro de   ABCD , como los tri´angulos equil´ ateros construidos son todos iguales (tienen lados iguales a la diagonal de  ABCD ) de altura h, OA = OB  =  OC  =  OD = h. Entonces, A B  C  D   es un cuadril´ atero con diagones que se bisecan en O  (es paralelogramo), son     iguales A C  = B D = 2h   (es rect´angulo) y son perpendiculares (es rombo), lo cual implica que es cuadrado.

 ⊥

80

5. Un trapecio is´osceles tiene diagonales perpendiculares y su ´area es 2010, determine su altura.

Soluci´ on:  Considere la figura 110. Sea   ABCD  el trapecio del problema ( AB = CD), como es trapecio is´osceles, es sim´etrico con respecto a la mediatriz de las bases, en particular, AC  =  B D . Sean P  y Q  los pies de las perpendiculares a AD   trazadas desde B y C , respectivamente. Por LAL, ABD DCA  lo cual implica ∠CAD  = ∠BDA  = 45 (debido a que AC  BD ) luego en el tri´angulo rect´angulo ACQ, ∠ACQ  = 45 por lo que AQ  =  C Q  y es f´acil ver que  B C  =  P Q. Luego

 ⊥

 

 ≡ 

 

(CQ)(BC  + AD) 2 (CQ)(P Q + AP  + P Q + QD) = 2 = (CQ)(AP  + P Q) = CQ2

2010 = (ABCD ) =

Luego  C Q  =

√ 2010.

Figura 110 6. (IX Competencia de Clubes Cabri, Segunda Ronda) Sea  ABCDEF   un hex´agono regular cuyo centro es O. Se construyen los cuadrados  FSOP  y  ORCQ. Demuestre que  APQB y  SEDR  son rect´angulos. Figura 111.

Soluci´ on:   Por construcci´on P F  = P O = SF  = SO , y por propiedades de hex´agono regular34 AF  =  AO  =  E F  =  EO , entonces P ,  S A,  E , pertenecen a la mediatriz de F O y por tanto, est´an alineados sobre una recta perpendicular a F O . An´alogamente, Q, R, B , D   est´ an alineados sobre una recta perpendicular a CO; adem´a s, es f´acil demostrar que AP  = BQ = DR = ES . Observe adem´as que AB CF  DE , lo cual implica (AB CF ) (AP  BQ ), es decir,  APQB  es rect´angulo, y an´alogamente para  SEDR.

 

34





 

Los tri´angulos  OAB ,  OBC ,  OCD ,  ODE ,  OEF ,  OF A  son equil´ateros.

81



Figura 111 7. Sobre los lados del ABC    se trazan exteriormente los cuadrados   ABPQ,   CARS  y BCTU . Luego se trazan los paralelogramos AQA  R,  C SC  T  y  BU B  P , como en la figura 112.



a ) Sean A , B  , C   los centros de los cuadrados  BCTU ,  CARS ,  ABPQ, respectivamente. Demuestre que estos centros est´an sobre los lados del A B  C  . b ) Demuestre que AA ,  B B  , CC   concurren.

 

Soluci´ on: a) Observe que al hacer una rotaci´on de centro A y ´angulo igual a 90, el A U B  se transforma en el A BA , y a la vez este ´ultimo se transforma en el A CC   (esto es porque A U  A B A C  y UB  BA CC  ); esto significa que A B  A A y A A  A  C  , por lo que B  , A , C   est´an alineados, es decir, A  pertenece a B  C  . An´alogamente se prueban los otros casos. b) De lo anterior, observe que AA   es mediatriz de B  C  , por lo que AA , BB  , CC  concurren en el circuncentro del A B  C  .

 ⊥

 →



 →

 →

 →



 

 ⊥

 

8. Se dibujan cuadrados exteriores a los lados de un paralelogramo (Vea figura 113), demuestre que: atero determinado por los centros de esos cuadrados es un cuadrado. a ) El cuadril´ b ) Las diagonales de ese cuadrado son concurrentes con las del paralelogramo.

Soluci´ on:

82

Figura 112 on de centro O2 y ´angulo igual a 90, el O2 BO 1 se a ) Observe que al hacer una rotaci´ transforma en el O2 CO3  (observe que los segmentos O2 B y BO 1   se transforman en O2 C  y CO3 , respectivamente), por lo que O2 O1 = O2 O3 y O2 O1 O2 O3 . Repitiendo este razonamiento, O1 O2 = O2 O3 = O3 O4 = O4 O1  y estos segmentos son perpendiculares si son consecutivos, por lo que O1 O2 O3 O4  es un cuadrado.

   ⊥

 

b ) Basta demostrar que AC  y O1 O3   se bisecan,35 y esto es equivalente a demostrar que AO1 CO3  es un paralelogramo. Esto es cierto porque AO1  =  C O3 y AO1 CO3 (ambos segmentos son perpendiculares a O1 B )

 

Figura 113 35

Porque as´ı los puntos de corte de las diagonales de  ABCD y  O 1 O2 O3 O4  coincidir´ıan.

83

9. Dado un ABC , se construyen exteriormente los tri´angulos rect´angulo is´ osceles ACP  y BC Q, con AC  y BC  como hipotenusas. Si M  es el punto medio de AB , demuestre que el M P Q  tambi´en es un tri´angulo rect´angulo is´ osceles.

     

 

Soluci´ on:  Construya los cuadrados exteriores  ACDE  y  BCFG, como muestra la figura 114. Observe que P  y Q  son los puntos medios de AD y BF , respectivamente. Al rotar el BC D  con centro C  y ´angulo de 90, se genera el F CA, entonces dichos tri´angulos son congruentes y en por tanto  B D  =  AF  y  B D AF . Por otra parte, observe que  M P  es base media del BAD , por lo que 2 M P  =  B D y M P   B D ; an´alogamente, M Q es base media del ABF , por lo que 2 M Q  =  AF  y  M Q AF . Por lo tanto M P  =  M Q  y M P  M Q.

 

 ⊥



   

 



Figura 114

84

 

´ Angulos en Circunferencia. 1. Dada la figura 115, demuestre que  AB

  AB.

Figura 115

Soluci´ on:   Observe que los cuadril´ateros   ABQP  y A B  QP  son c´ıclicos, por lo que    A B  . ∠P AB = ∠P QB  = 180 ∠P A B , por lo tanto AB

 −

 

2. Dos circunferencias de centros O1 y O2   son tangentes (interna o externamente) en un punto P ; por este punto se traza una recta que corta nuevamente a la circunferencias en A  y B , respectivamente. Demuestre que AO1 BO 2 .

 

Soluci´ on:  En la figura 116 se ha considerado que las circunferencias son tangentes exteriormente, sin embargo, el otro caso es an´alogo. Se sabe que O 1 ,  P ,  O 2  est´an alineados, y que AP O1 y  B P O2  son tri´angulos is´ osceles (dos de sus lados son radios de una circunferencia), entonces ∠O1 AP  = ∠AP O1  = ∠BP O2  = ∠O2 BP , por lo que AO1 BO 2 .

 

 

Figura 116 3. Dadas dos circunferencias una fuera de la otra como en la figura 117, demuestre que las tangentes comunes externas forman segmentos iguales; an´alogamente, las tangentes comunes internas forman segmentos iguales.

Soluci´ on:36 Sea P  la intersecci´on de las tangentes comunes externas  AA   y  BB  . Entonces 36

Suponemos que las circunferencias tienen radios distintos; cuando los radios son iguales, el problema se  justifica por la simetr´ıa de la figura.

85

AA  =  P A

− P A =  P B  − P B  =  BB . An´alogamente se resuelve el caso de las tangentes

comunes internas.

Figura 117 4.   Teorema de Pithot.  Demuestre que en todo cuadril´atero inscribible, la suma de lados opuestos es igual.37

Soluci´ on:  Considere la figura 118,  ABCD  es el cuadril´atero inscribible, con  P ,  Q ,  R ,  S , los puntos de tangencia sobre AB ,  B C , CD, DA , respectivamente. Entonces AB + CD = AP  + P B  + CR + RD

= AS  + BQ + CQ + DS  = BC  + DA

Figura 118: Teorema de Pithot. 5.  Teorema de Steiner.  En todo cuadril´atero exinscrito a una circunferencia, la diferencia de las longitudes de lados opuestos es igual.

Soluci´ on:  El cuadril´atero puede quedar en posiciones como las de la figura  ?? ; en ambos casos, la demostraci´o n es muy similar, y an´a loga a la de Pithot. Para la figura de la 37

El rec´ıproco de este teorema y del siguiente son tambi´ en es ciertos.

86

izquierda se tiene que AB

− CD

= (AP  BP ) = (AS  BQ ) = AD BC 

 − − −

− (CR − RD) − (CQ − DS )

Figura 119: Teorema de Steiner. 6.   Teorema de Miquel:   Dado un ABC , sean X , Y  , Z   puntos sobre AB , BC , CA, respectivamente . Demuestre que los circunc´ırculos de AXZ , BY X , CZY   tienen un punto en com´ un M .

 

 

Soluci´ on: Sea  M  el otro punto de corte de los circunc´ırculos de los cuadril´ ateros  AXMZ  y  BY M X   son c´ıclicos, se tiene ∠Y

 

 

AXZ  y BY X . Como

MZ    = 360

= =

− ∠XM Z  − ∠Y M X  360 − (180 − ∠A) − (180 − ∠B ) 180 − ∠C 

Entonces,   CY M Z   es cuadril´atero c´ıclico, por lo que M   est´a sobre el circunc´ırculo del CZY .



Figura 120: Teorema de Miquel.

87

7. Sea ABC  un tri´angulo, y sean L y N  las intersecciones de la bisectriz del ´angulo A con el lado BC  y el circunc´ırculo de ABC  respectivamente (Ver figura 121). Construimos la intersecci´on  M  del circunc´ırculo de  ABL  con el segmento  AC . Prueba que los tri´angulos BM N  y  B M C  tienen la misma ´area.

Soluci´ on:  Observe que  ABNC  y  ABLM  son cuadril´ateros c´ıclicos, por lo que ∠N CB = ∠N AB  = ∠LAM  = ∠LBM , por lo que  C N  BM . Entonces, las distancias de  N  y  C  a la recta BM  son iguales, y por tanto, el ´area del BM N  es igual al a´rea del BM C .

 

 

 

Figura 121 8. Sea AB   el di´ametro de una semicircunferencia. Se colocan los puntos M  y K   sobre la semicircunferencia y sobre AB , respectivamente.38 Sea P   el centro de la circunferencia que pasa por A, K  y M ; sea Q  el centro de la circunferencia que pasa por B , K  y M . Demuestre que  M P K Q  es conc´ıclico.

Soluci´ on:  Como  AB  es di´ametro, ∠AMB  = 90, entonces ∠M P K  + ∠MQK 

= 2∠MAK  + 2 ∠MBK  2 = 2 (∠M AB + ∠M BA ) = 180

Por lo tanto,  M P K Q  es conc´ıclico.

38

∠M AB  + ∠MBA  =

M  y  K  son distintos de  A  y  B .

88

90. As´ı

9. Las circunferencias Γ1  y Γ 2  se cortan en los puntos A y B . Por el punto A se traza una recta que corta nuevamente a las circunferencias Γ 1  y Γ2  en los puntos  C  y  D , respectivamente. Por los puntos C  y  D  se trazan tangentes a las circunferencias, las cuales se cortan en el punto  M . Demuestra que  MCBD  es c´ıclico. Figura 122.

Soluci´ on:   Es suficiente probar que   MCBD  es un cuadril´a tero con un par de ´angulos opuestos suplementarios. Por a´ngulos seminscritos y suma de a´ngulos internos de un tri´angulo, se tiene ∠CM D  + ∠CBD

= ∠CM D + ∠CBA + ∠DBA = ∠CM D + ∠M CA + ∠M DA = 180

Figura 122 10. El ABC  cumple que ∠A  = 90 y  AB  =  AC . Se toma un punto  E  del segmento  AB , se construye interiormente un tri´angulo equil´atero  AE F .  E F   corta  BC  en  I , y se construye exteriormente un tri´angulo equil´atero  B IJ . Encuentre ∠EJ B .



Soluci´ on:   Como el ∠BJ I  = 60◦ = ∠AEI , el cuadril´atero  BEIJ   es c´ıclico, por lo que el ∠EJ B  = ∠EI B  = ∠AEI  ∠EB I  = 15 .



89

11. En la figura 123, se sabe que ∠AO1 B ∠AO2 B = 70◦  y adem´as la tangente EB  forma el tri´angulo is´ osceles  AB E , con AB  =  AE . Encuentre ∠EBC .



Figura 123

Soluci´ on:  Sea ∠AO2 B  = 2θ , entonces ∠ACB  =  θ  y por hip´oteis ∠AO1 B  = 2θ + 70. Por ´angulo seminscrito, ∠ABE  = θ  + 35, y como el ABE   es is´osceles, ∠AEB = θ  + 35. Finalmente, por la f´ormula del a´ngulo externo aplicada al BC E , ∠EB C  = ∠AEB ∠EC B  = 35

 

 



12. Dos circunferencias Γ1 y Γ2  se cortan en  A  y  B . Una recta por A  corta a Γ1 y Γ2 en  C  y D, respectivamente, y la paralela a CD por B  corta Γ1 y Γ2 en E  y F , respectivamente. EAF . (Ver figura 124). Demuestre que CDB

 

 ≡  Soluci´ on:  Sean G = AE  ∩ BC  y H  = AF  ∩ BD . Como AC   BE  y  ACEB  es c´ıclico, ∠CAG = ∠GEB = ∠ACG = ∠GBE  = α; an´alogamente, ∠DAH  = ∠HF B = angulos is´ osceles y por tan∠ADH  = ∠F BH  =  β . Observe que GAC  y GBE  son tri´ to AE  =  AG +GE  =  C G+GB  =  C B ; de forma similar se obtiene AF  =  DB . Finalmente, ∠EAF  = 180 − α − β  = ∠CBD , por lo que, por el criterio LAL,  CBD ≡ EAF . 39

13.  La Recta de Simson-Wallace.  Sean  X ,  Y  y  Z  los pies de las alturas trazadas desde un punto  P  en el circunc´ırculo del ABC  hacia  AB ,  BC  y  C A, respectivamente. Demuestre que X , Y  y  Z  est´an alineados.40



Soluci´ on:   Como   BP Y X   es c´ıclico, ∠Y XP  = Y BP  = θ . Como   ABPC   es c´ıclico, ıclico, ∠ZAP  = ∠ZXP  = θ . Por lo tanto, ∠CBP  = ∠CAP  = θ . Como   AXPZ  es c´ dado que ∠Y XP  =  Z XP , los puntos X , Y  y Z  est´an alineados. 39

ACEB  es un trapecio is´osceles.

40

El rec´ıproco tambi´en es cierto, si X , Y  y Z   est´an alineados, entonces P   debe estar sobre el circunc´ırculo

del

 ABC ; en cualquier otro caso, el  XY Z   se llama el  tri´ angulo pedal  con respecto al punto  P . 90

Figura 124

Figura 125: Recta de Simson-Wallace 14. Sea P  un punto exterior al cuadrado  ABCD  tal que ∠AP C  = 90◦ , Q  es la intersecci´on de  AB y  P C , y  R  el pie de la perpendicular por  Q  a  C A. Demuestre que  P ,  R  y  D  est´an alineados.

Soluci´ on:   Como ∠AP C  + ∠ADC   = 180  , el cuadril´atero   PADC   es c´ıclico, entonces alogamente, como ∠AP Q  + ∠ARQ  = 180 , el cuadril´atero ∠AP D = ∠ACD = 45  . An´ PARQ   es c´ıclico, entonces ∠AP R = ∠AQR = 90 ∠QAR = 45 . Por lo tanto, como an alineados. Figura 126. ∠AP D  = ∠AP R, los puntos P ,  R ,  D  est´

 −

15. (OIM 2002, P-4) En un tri´ angulo escaleno  AB C  se traza la bisectriz interior  B D , con  D sobre  AC . Sean  E  y  F  puntos sobre la recta  BD  tales que ( AE  CF ) BD , y sea  M  el punto sobre el lado BC  tal que DM  BC . Demuestre que ∠EM D = ∠DM F . Figura 127.

 

 ⊥

91



Figura 126

Soluci´ on:  Como DM  M C  y DF  F C ,  DFCM   es c´ıclico, por lo tanto ∠DM F  = F C , entonces ∠EAD = ∠DCF  = θ . Sea G   la intersecci´on ∠DCF  = θ , y como AE  de AE  con BC . Como AG BE , BE  es altura y bisectriz del ABG , por lo que este tri´angulo es is´osceles y adem´as  B E  es mediatriz de AG; entonces ∠EGD  = ∠EAD  =  θ . Y finalmente, podemos ver que  DEMG  es c´ıclico, pues ∠DEG  = ∠DM G  = 90◦ , as´ı que ı, el resultado es inmediato. ∠EM D  = ∠EGD  =  θ . De aqu´



   ⊥



 

Figura 127 16. (OMCC 2003, P-2) Sea S  una circunferencia y AB   un di´ametro de ella. Sea t  la recta tangente a  S  en  B  y considere dos puntos  C  y  D en  t  tales que  B  este entre  C  y  D . Sean E  y F  las intersecciones de S  con AC  y AD  y sean G y H  las intersecciones de S  con CF  y  DE . Demuestre que AH  =  AG .

Soluci´ on:  Como  AEBF  es c´ıclico (Ver figura 128), ∠AEF  =  ABF . Luego, como  AB CD y BF  AD, se cumple tambi´en ∠ABF  = ∠F DB , por lo que ∠AEF  = ∠F DC , es decir, el cuadril´atero  CDFE   es c´ıclico. Utilizando este resultado y el hecho que  EGHF  tambi´en es c´ıclico, se tiene ∠EDC  = ∠EF G = ∠EH G, por lo que CD GH . Esto

 ⊥

 ⊥



92

implica que AB GH , y como AB   pasa por el centro de la circunferencia, debe ser mediatriz de GH , por lo tanto AG  =  AH.



Figura 128 17. (The 59th Romanian Mathematical Olympiad District Round) Considere un cuadrado ABCD  y un punto  E  sobre el lado  AB . La diagonal  AC  corta al segmento  DE   en el punto  P . La perpendicular por  P  a  DE   corta al lado  BC  en  F . Probar que  E F  =  AE  + CF .

Soluci´ on:  Se construye E    sobre BC   de tal manera que CE  = AE   (como se muestra en la figura 129) y que  C  quede entre  F  y  E  , as´ı por LAL se tiene que los tri´angulos rect´angulos DAE  DCE   por lo tanto ∠ADE  = ∠CDE   luego ∠EDE   = 90. Por otra parte, el cuadril´atero  DCFP   es c´ıclico, por lo que ∠P DF  = ∠P CF  = 45 entonces   ∠EDF   = 45. Ahora por LAL los tri´angulos DEF  DE  F , ∠F DE  = ∠EDE  por lo que  E F  =  E  F  =  E  C  + CF  =  AE  + CF .

 

 ≡  −

 

Figura 129

93

≡

18.   Teorema de Arqu´ımedes:  En la figura 130, la regi´on delimitada por tres semicircunfearbelos . Demostrar rencias mutuamente tangentes, es conocida como cuchilla de zapatero  o  ´  que las circunferencias sombreadas son congruentes.

Figura 130: Teorema de Arqu´ımedes.

Soluci´ on: Sean  AB ,  AC ,  BC   los di´ametros de las semicircunferencias que forman el ´arbelos, de radios r , r1 , r2  y centros O, O1 , O2 , respectivamente. De momento nos concentramos en el lado izquiero de la figura Referenciasfigura58; sea  C 1  el centro de la circunferencia de la izquierda y  R1  su radio;  D , E  y  F  son los puntos de tangencia de esta circunferencia con dos semicircunferencias del ´arbelos y con la recta perpendicular a los di´ametros por C ; finalmente, G  es la proyecci´on de C 1  sobre AB . En primer lugar,  OO 1  =  OA O1 A  =  r r1 . Por otra parte, observe que  O ,  C 1 ,  D  est´an alineados y O 1 ,  C 1 ,  E  tambi´en est´an alineados, entonces OC 1 = OD C 1 D = r R1 y O1 C 1 = O1 E  +  EC 1 =  r 1  +  R1 . Adem´as, como CF C 1 G  es un rect´angulo, GC  = F C 1 = R1 , entonces O1 G = O1 C  GC  = r1 R1 y OG  =  O 1 G O1 O  = (r1 R1 ) (r r1 ) = 2r1 (r + R1 ). Ahora, aplicando el teorema de Pit´agoras a GO1 C 1 y GOC 1  se tiene









− − −     O C   − O G − C  G (r  + R ) − (r − R ) 1

2 1

2

1

2

1

2

1

1

2

1

1

4r1 R1

⇒R

1



 −





= OC 12 OG 2 C 1 G2 = (r R1 )2 (2r1 (r + R1 ))2 = 4rR1 4r12 + 4 r1 r + 4 r1 R1 r1 (r r1 ) =

 − − − − − − − − r

Figura 131 An´alogamente, si r2 y R2  son los radios del semic´ırculo y del c´ırculo de la derecha, respectivamente, entonces r2 (r r2 ) R2  =



r

94

Pero 2r = AB = AC  +  BC  = 2r1 + 2r2 , entonces r2 = r ecuaci´ on anterior se tiene R2  =

(r

− r ) (r − (r − r )) = (r − r )r 1

1

r

1

r

95

 − r , y sustituyendo en la

1

1

=  R 1

Teorema de Thales y Semejanza. 1. (IV OMCC, P-4) Sea ABC   un tri´angulo, D  el punto medio de BC , E   un punto sobre el segmento AC   tal que BE  = 2AD y F   el punto de intersecci´o n de AD con BE . Si angulos del F EA . Figura 132. ∠CAD  = 60 , encuentre la medida de los ´

 

Soluci´ on:  Se traza por D  una paralela a BE  y sea G  el punto por el que esta paralela corta al lado AC . Como DG  es base media del BC E  se tiene que DG = BE  = AD ; 2 entonces ADG   es is´o sceles y tiene un ´a gulo de 60, por lo que debe ser equil´atero. Finalmente AEF   tambi´en es equil´atero, por tanto sus ´angulos son iguales a 60.

 

  

Figura 132 2. Sea  ABCD  es un trapecio con AD BC . M  y N  son los puntos medios de CD y BC , M  respectivamente, y  P  el punto com´un de las rectas  AM  y  DN . Si PAP  = 41 , demuestre que ABCD  es paralelogramo.

 

Soluci´ on: Sea Q   el punto medio de DN , entonces QM  BC  DA . Como M Q es base media del CDN , M Q = CN  = CB   . Por otra parte, como P MQ P AD, 2 4 MQ 1 M  = PAP  = 4 , entonces M Q  = AD   . Finalmente, como BC   DA y BC  =  DA,  ABCD AD 4 es paralelogramo.

  

 

3. En la figura 133, BC  =  C D  =  DE  =  E A  =  x  y ∠ACD + ∠ADE  = 90  .

∠AEB  =

  

  

90  . Demuestre que

∠ABC  +

Figura 133

√ 

Soluci´ on 1:  Por Pit´agoras,  AD  = 2x. Observe que  DA2 = 2 x2 =  DB DC , por lo que ABD CAD; entonces ∠ABD  = ∠CAD  y por tanto ∠ABC  + ∠ACD + ∠ADE  =



·



96

∠CAD + ∠ACD  + ∠ADE  =

2∠ADE   = 90.

Soluci´ o n 2:   Considere la siguiente cuadr´ıcula (Figura 134). Observe que al hacer una rotaci´on de centro A y ´angulo igual a 90, el segmento AC  se transforma en AF , por lo que el ACF  es tri´angulo rect´angulo is´ osceles, y ∠ADE  = ∠ACF . Se cumple ∠ABC  = alogamente, ∠ACD = ∠CF D , porque se forman con la diagonal de tres cuadrados; an´ angulos internos ∠AF D , porque se forman con la diagonal de dos cuadrados. Sumando los ´ del ACF  se obtiene el resultado buscado.



 

Figura 134 4. (Asi´atico Pac´ıfica) Sea  ABC   un tri´angulo y D  el pie de la altura con respecto a  A. Sean  E  y F  puntos en una recta que pasa por  D  (distintos de D ) tales que  AE  CE  y  AF  BF . Sean M  y N  los puntos medios de BC  y  EF , respectivamente. Demuestre que  AN  N M .

 ⊥

 ⊥  ⊥

Soluci´ on:  En la figura 135, AE  CE  y AD DC  entonces,  ADEC   es c´ıclico, as´ı que BF  y AD DB ,   AFBD  es c´ıclico ∠DEA = ∠DCA. Del mismo modo, como AF  AF E   y a partir de esta y entonces ∠AF D = ∠ABD . Esto implica que ABC  semejanza, ABM  AF N . Luego, ∠AMB = ∠AN F , por lo que el cuadril´atero ANDM   es c´ıclico, y por lo tanto ∠AN M  = ∠ADM  = 90.

 ⊥

 

 ⊥



97

⊥  

 ⊥ 

Figura 135

Puntos y rectas notables. 1. De acuerdo con los datos de la gr´afica 136, calcular el valor de  AB .

Figura 136 AC  Soluci´ on 1: Por el teorema de la bisectr´ız AB = CD , de donde AC  = 45 x, luego, aplicando DB 2 el teorema de pit´agoras al ABC , se tiene que  x 2 + 182 = 54 x , que despu´es de resolver se tiene que x  = 24.





Soluci´ o n 2:  Dado que D  es un punto de la bisectr´ız del ∠BAC , entonces D  equidista de los lados de dicho ´angulo, sea pues H  AC   talque DH  AC  y DH  = DB = 8 entonces, aplicando el teorema de pit´agoras en el CDH  se deduce que HC   = 6, por lo que  AC  =  x + 6, y por el teorema de pit´agoras en el ABC ,  x 2 + 182 = (x + 6)2 , por lo tanto, x  = 24.



 



 

2. Sea  ABCD   un paralelogramo. Q  es el punto medio de  AD,F  el pie de la perpendicular por B  sobre QC . Probar que AF  =  AB . Soluci´ on: Sea E   el punto medio de BC  y G  la intersecci´o n de AE  con BF . Como AE  CQ, se tiene que AG BF . Pero tambi´en, como AE  CQ, entonces EG CF  por lo que en el BC F , EG  es base media. Entonces BG  =  GF  de donde se sigue que ABF  es is´ osceles porque  B G  es altura y mediana.

 



 

 ⊥

 

98

 

3. Sea  ABCD  un cuadril´atero tal que AB = CD. Las mediatrices de AC  y BD  se cortan en  P . Probar que ∠P AC  = ∠P CA  = ∠P BD  = ∠P DB . Figura 137. Soluci´ on:  Como P   est´a sobre las mediatrices de AC  y BD , P A = P C  y P B = P D , CDP . De aqu´ı, y por hip´otesis, AB = CD, entonces por criterio LLL, ABP  ∠AP B = ∠CP D , entonces ∠BP D = ∠AP D  + ∠AP B = ∠AP D  + ∠CDP  = ∠AP C ; CP A, dada la igualdad anterior y el hecho que son tri´angulos por lo tanto, BP D is´osceles. De esta semejanza se obtiene ∠P AC  = ∠P CA  = ∠P BD  = ∠P DB .

 

 

≡

  

Figura 137 4. ABC   es un tri´angulo y P  un punto en su interior. Sean A , B  y C    las reflexiones de P   sobre BC , CA y AB , respectivamente. D , E  y F   son los pies de las perpendiculares respectivos desde A, B y C   hacia B  C  , C  A y A B  . Probar que AD , BE  y CF  son concurrentes. Figura 138.

Soluci´ on:  Por propiedades de reflexi´on axial  AC   =  AP  =  AB  , por lo que el AB  C   es is´osceles, y entonces  AD  es mediatriz de B  C  . An´alogamente, BE  es mediatriz de C  A , mientras que CF  es mediatriz de A B  . Por lo tanto, las rectas AD , BE , CF   concurren en el circuncentro del A B  C  .



 

5. (Arnoldo Aguilar) En la figura 139,  ABGH ,  BCFG y  CDEF  son cuadrados. Si I   es el centro de  ABGH  y J  =  DH  BG , demuestre que I , J  y F   est´an alineados.



Soluci´ on:  Como G   es punto medio de HF , BG  es una mediana del BF H . Adem´as, BDFH   es un paralelogramo, luego sus diagonales BF  y  DH  se cortan en su punto medio, digamos K . Se sigue que HK  es tambi´en una mediana del BF H , y en consecuencia el punto de corte de  J  =  K H  BG  es el centroide del BF H . Pero  I  es el punto medio de BH , as´ı que  F I  es la tercera mediana del BF H , por lo tanto  J  est´ a sobre el segmento F I .



 

 

99



 

Figura 138

Figura 139 6. (Arnoldo Aguilar) Sea ABC   un tri´angulo equil´atero. M  y  N  son los puntos medios de  AB y BC , respectivamente. Exteriormente al ABC   se construye un tri´angulo rect´angulo is´osceles AP C , con ∠AP C  = 90◦ . Si  I  es la intersecci´on de AN  y MP , demuestre que CI  es la bisectriz de ∠ACM .

 

 

Soluci´ on:  Observe que AN  es bisectriz del ∠BAC . Como ∠AP C  = ∠BM C  = 90, el cuadril´atero APCM   es c´ıclico, por lo que ∠P M C  = ∠P AC  = ∠P CA = ∠P M A   = 45, entonces M P   es bisectriz del ∠AMC . De aqu´ı se concluye que  I  =  M P  AN  es el incentro del ACM , por lo que CI  es bisectriz del ∠ACM .

 ∩

 

7. En la figura 140, el circunferencia?

 ABC   es tal que ∠A   = 90 y ∠B   = 60. ¿Cu´al es el radio de la √  Soluci´ on:  Por relaciones de tri´angulos notables, BC  = 2 y CA = 3. Sean P  y Q las proyecciones de O  sobre AB y AC   respectivamente; por construcci´on,  APOQ  es un rect´angulo, pero como  OP  =  r  =  OQ, es tambi´en cuadrado, por lo que  AP  =  r . Observe 100

Figura 140 que la circunferencia es el exc´ırculo del

 ABC , por lo que  AP  = s  y entonces

r = s

1+2+ = 2 3+ 3 = 2

√ 3

√ 

8. Dado el paralelogramo ABCD , sea  M  el punto medio de  AB , y  N  la intersecci´on de  C D con la bisectriz interna del ∠ABC . Demuestre que  M C  BN  si y s´olo si  AN  es bisectriz del ∠DAB .

 ⊥

Soluci´ on: ( ) Si suponemos que  M C  BN   entonces  BN  es mediatriz de  M C , y como  BM  CN  entonces ∠CBN  = ∠MBN  = ∠CN B = ∠MN B , esto implica que BC  M N , y por tanto  N  es punto medio de  C D; as´ı,  AMN D  es un rombo y  AN  es bisectriz del ∠DAM . ( ) Si suponemos que  AN  es bisectriz del ∠DAB , es propiedad conocida que  AN  BN , por lo que M  es el circuncentro del ABN  y por la relaci´on entre ´angulo central y ´angulo inscrito se tiene ∠AMN  = 2∠ABN  = ∠ABC , por lo tanto MN  BC  y  BCNM   es un rombo, de donde se obtiene MC  BN .



 ⊥



9. En el

 



 ⊥

  ⊥

 

 ABC , se sabe que los v´ertices B , C , el circuncentro O   y el ortocentro H  del

ABC  est´an todos sobre una misma circunferencia. Figura 141. a) Calcule el valor de ∠A.

b) Demuestre que el incentro tambi´en pertenece al circunc´ırculo de  BCOH .

Soluci´ on: a) Sea ∠BAC  =  α . Como O  es el circuncentro del ABC , tenemos que ∠BOC  = 2α. Por otra parte, sabemos que al ser  H  ortocentro, se cumple que ∠BH C   = 180◦ α. Ahora



101



bien, la condici´on de que B ,  C , H  y  O  son conc´ıclicos implica que de donde 2α  = 180◦ α, y por tanto α  = 60◦ .

∠BOC  = ∠BH C ,



Figura 141 b) Este problema se basa en el siguiente resultado: si  I  es el incentro del ABC  entonces ∠A ∠BI C  = 90 + 2  . Como en este caso ∠A  = 60, entonces ∠BI C  = 120 = ∠BOC  = ∠BH C , por lo que B , C ,  O ,  H ,  I , se ubican sobre una misma circunferencia.



10. Sea ABC   un tri´angulo tal que las medianas respectivas a B y C  son perpendiculares. Demuestre que se cumple la relaci´on (Ver figura 142). 5BC 2 =  C A2 + AB 2 .

Figura 142

Soluci´ on:   Sean  B B  y  C C   las medianas que son perpendiculares, y sea  G  el centroide. Observe que el cuadril´atero BC B  C   tiene diagonales perpendiculares; por el teorema de Pit´agoras se cumple BC 2 + B  C 2 = C  B 2 + B  C 2 2 2 BC  AB AC  2 BC  + = +

      2 2 5BC 2 = AB 2 + AC 2 102

2

2

11. Sea  ABC   AB C  un   un tri´angulo angulo de ortocentro  H . Sean  P  y  Q  los pies de las perpendiculares desde H  a las bisectrices interior y exterior de A, respectivamente. Si M  es el punto medio de BC , mostrar que  P ,  Q y M   est´ an alineados. Figura 143. an

Soluci´ on: on:   Sean E  y  F  los pies de las alturas trazadas desde a  B y  C , respectivamente. Se sabe que AP  AQ, por lo que  AP  AP HQ  es un rect´angulo. angulo. Como ∠AP H  = ∠AQH  = ∠AEH  = ∠AF H   = 90, los puntos P , Q, E , F , pertenecen a una circunferencia de di´ametro ametro AH . Adem´as, as, en esta circunferencia, como AP  y AQ   son bisectrices (interior y exterior, respectivamente) del ∠EAF  , P  y Q  son los puntos medios del arcos EF , por lo que P Q  es la mediatriz de EF . Por otra parte, como ∠BE C  = ∠BF C  = 90, el cuadril´atero atero  BCEF  es c´ıclico, y el e l circuncentro c ircuncentro es  M , por lo que  M E  =  =  M F ; entones  M  est´ a en la mediatriz de  E F , la cual es P Q.

 ⊥  ⊥

Figura 143 12. En un tri´angulo angulo ABC , sea M   el punto medio de BC . Si se cumple que AB = AC  y adem´as as ∠M AC  +  + ∠ABC  = 90 ◦ , hallar ∠BAC . Figura 144.



Soluci´ on: on:  Sin p´erdida erdida de generalidad, suponga que AB > AC . Sea N  la intersecci´on on de AB   con la mediatriz de BC . Se forma el BC N   que es is´osceles, osceles, entonces ∠C N M  = 90 ∠M C N  = 90 ∠M BN  = ∠C AN , lo cual implica que el cuadril´atero atero  ACMN  es c´ıclico. ıcli co. Por lo tanto, tanto , ∠BAC  = ∠BM N  = 90.



 



13. Sea ABC   un tri´angulo angulo y U   un punto de su circunc´ circunc´ırculo tal que AU   es bisectriz. Las mediatrices en AB y AC  cortan a AU  en X  y Y  . Sea T   la intersecci´on on de BX  con C Y  . Demostrar que AU  =  T B  + T C . Figura 145. Soluci´ on: on:41 Como X  y Y   pertenecen a las mediatrices de AB y AC , respectivamente, ACY  Y  cumplen ser is´osceles y a la bisectriz AU , entonces ABX  y AC osceles y semejantes entre si, porque ∠X BA  = ∠X AB = ∠Y AC  = ∠Y C A  =  α . Esto implica ∠T X Y  = ∠X BA  +

 

41

 

El caso cuando  AB  =  AC  es trivial, porque  X ,  Y  y  T  colapsan en el circuncentro del

103

 ABC .

Figura 144 = 2 α  = ∠Y AC +∠Y C A  = ∠T Y X , es decir, el T X Y  es is´osceles osceles con T X  =  T Y  . Por otra parte, como  ABUC   es c´ıcli ıc lico co,, ∠U BC  = ∠U AC  = ∠U AB = ∠U C B = α. De aqu´ı se concluye con cluye que U BC  es is´osceles, osceles, con U B =  U C . Adem´as, as, ∠X U B = ∠ACB = U XB C Y U , por lo que ∠Y C U  y ∠X BU  = ∠ABC  = Y U C ; por criterio ALA, BX  =  Y U . Finalmente,  T B  + T C  = (BX  T X ) + ( C Y  + T Y  ) =  Y U  +  + AY  =  AU .



∠X AB

 

 



 ≡ 

Figura 145 14. (The 59th Romanian Mathematical Olympiad Final Round) Sea  ABCD  un rect´angulo angulo de centro  O con  AB =  BC   B C . La perpendicular en  O  a  BD  B D  corta  cor ta a las l´ıneas ınea s  AB y  B C  en   en los puntos E  y F , respectivamente. Sean M  y N  los puntos medios de los segmentos C D y DA, respectivamente. res pectivamente. Probar que las l as l´ıneas ıneas rectas  F M  EN .

 

 ⊥  ⊥

Soluci´ on: on: Considere la figura 146, sin s in p´erdida erdida de generalidad, generali dad, se ha h a supuesto supues to AB < BC .42 Sea L   el punto medio de AB , y H   es la intersecci´on o n de EF  con AD. Se tiene que LN  BD , y como  BD  B D EF   entonces  LN  EF ; adem´ as, as, como  ABCD  es un rect´anguangulo, DA AB , por lo tanto, H  es el ortocentro del ELN , y as´ı, LH  EN . Por otra parte, las reflexiones de L y H  con respecto a O  son respectivamente  M  y  F , por lo que LH  M F , lo cual implica que F M  EN .

  

 ⊥



  

42

 ⊥  ⊥

 ⊥  ⊥

El otro caso es completam completament entee an´alogo. alogo.

104

 

 ⊥  ⊥

Figura 146 15. Sea ABC   un tri´angulo angulo rect´angulo, angulo, con A = 90◦ . Sea D  un punto en su interior tal que DCA A  = ∠DBC  DB C  =  α , y  AC  =  BD  B D. Determine el valor de α. Figura 147. ∠DAC  = ∠DC

Soluci´ on: on:  Sean  P  y  Q  los pies de las perpendiculares trazadas desde  D  hacia  C A  y  AB , respectivamente,  R es un punto sobre  BC   B C  tal   tal que  DB DR , y  E  es  es la intersecci´on on de CD  C D ACD D  es is´ con AB . Como el AC osceles, osceles, P  es punto medio de AC , entonces AC  = 2P A  = 2DQ  =  BD  B D, por lo que el BDQ BD Q  es un tri´angulo angulo notable y ∠DBQ DB Q  = 30. Por otra parte, DBR DB R, por lo que  C E  =  BR  B R; como  P D AE ,  D  es punto por criterio ALA, ACE  medio de  C E ; as´ı, si  M  es el punto medio de  B R  (y circuncentro del BDR BD R) se cumple  D C  =  RM  =  DM , por lo que el C DM  es is´ que  DC  osceles. osceles. Por la relaci´on on entre el ´angulo angulo inscrito y el ´angulo angulo central ∠DM R  = 2∠DBR DB R, por lo tanto ∠DC DCR R  = 2α. Sumando los angulos ´angulos internos del ABC  se tiene ∠A + ∠B  + ∠C  =  = 90 + 30 + α + 3 α  = 180, lo cual implica  α  = 15.

   

 ⊥

  ≡   ≡

  



 

Figura 147 16. Sea ABC  un tri´angulo angulo y M  un punto tal que 105

∠M AB  =

10,

∠M BA  =

20,

∠M AC  =

40

y ∠M CA  = 30. Probar que el ABC  es is´osceles. Figura 148. Soluci´ on: Sea D  la reflexi´on del punto A con respecto a la recta  BM . Entonces el AM D es is´osceles con ∠AM D  = 2 (∠M AB + ∠ABM ) = 60 y por lo tanto es equil´atero. TamAC . Sea E  la bi´en ∠DBA = 2∠MBA  = 40 y como ∠BAC  = 50, implica que DB intersecci´o n de BD con CM , se cumple que ∠CED = 90 ∠ACE   = 60 = ∠MAD, por lo que el cuadril´atero   AMED  es c´ıclico. De aqu´ı, ∠DEA = ∠DM A  = 60. Como AC , se tiene que ED  es bisectriz y altura en el AEC , por ∠DEC  = ∠DEA y ED lo tanto ED  es mediatriz de  AC , lo cual implica que BA  =  BC .

 





 ⊥

 ⊥

 

Figura 148 17.  Teorema de Poncelet:  Demuestre si entonces 2(r + R) =  b + c.

 ABC  es un tri´angulo rect´angulo con ∠A = 90 ,

Figura 149: Teorema de Poncelet

Soluci´ on:  Sean O e  I  el circuncentro y el incentro del ABC . Como ∠A  = 90 , O  es el punto medio de  BC , por lo que  a  = 2R. Por otra parte, si  P  y  Q  son las proyecciones de  I  sobre  AB y  AC , claramente  AP IQ  es rect´angulo, pero como  I  es incentro  I P  =  r  =  I Q, por lo que  AP IQ  es cuadrado. Se sabe que para un tri´angulo cualquiera  AP  =  s a, por lo tanto

 



r = s a b+c r =



2

2r + a = b + c 2(r + R) = b + c 106

−a

18. En la figura 150, ABCD  y  P QRS   son cuadrados, ABP  BC Q CDR DAS  y los los radios de las cinco circunferencias son iguales a r . Si a  es el lado del cuadrado ABCD , determine  r  en funci´ on de a.



 ≡ 

≡

≡

Figura 150

Soluci´ on:  Se tiene  AB  =  a  y se definen  b  =  AP  y  c  =  BP ; observe que por las congruencias  BQ  =  b , por lo que  P Q  =  c b  = 2r. Por otra parte (an´alogamente a la demostraci´on del teorema de Poncelet), al calcular el inradio del ABP  se tiene que 2r = b  +  c a, entonces c b = b  +  c a, lo cual implica que a = 2b. Por lo tanto, el ABP   es un tri´angulo notable de 30, 60, 90, y as´ı





 



r =

= =

c

−b

√ 2 3 2



107

1 2

− a √ 32 − 1 a

4



a

 



View more...

Comments

Copyright ©2017 KUPDF Inc.
SUPPORT KUPDF